Sunteți pe pagina 1din 73

INTRODUCCIÓN A LA TERMODINÁMICA

Definiciones, Dimensiones y Unidades, Formulas Básicas, Sistema


Internacional de Unidades (SI) y Conversiones
Introducción a la termodinámica

Si buscamos una definición sencilla de termodinámica podemos encontrar que la


termodinámica es la rama de la física que estudia la energía, la transformación entre sus distintas
manifestaciones, como el calor, y su capacidad para producir un trabajo. La termodinámica está
íntimamente relacionada con la mecánica estadística, de la cual se pueden derivar numerosas
relaciones termodinámicas. Es importante tener en mente que la termodinámica estudia los
sistemas físicos a nivel macroscópico, mientras que la mecánica estadística suele hacer una
descripción microscópica de los mismos.

Qué se entiende por termodinámica

Debe quedar claro que la termodinámica es una ciencia y, quizá la herramienta más importante
en la ingeniería, ya que se encarga de describir los procesos que implican cambios en
temperatura, la transformación de la energía, y las relaciones entre el calor y el trabajo.

La termodinámica es una ciencia factual que se encarga de estudiar hechos o acontecimientos


auxiliándose de la observación y la experimentación por lo que tiene que apelar al examen de
la evidencia empírica para comprobarlos. Así, la termodinámica puede ser vista como la
generalización de una enorme cantidad de evidencia empírica.

Esta ciencia es extremadamente general: no hay hipótesis hechas referentes a la estructura y al


tipo de materia de la cual nos ocupamos. Tal vez una de las razones por las que la termodinámica
es tan difícil de estudiar sea que la teoría empleada para describir los fenómenos es muy general
y que puede ser aplicable a sistemas de estructura muy elaborada con todas las formas de
propiedades mecánicas, eléctricas y térmicas complejas.

En el estudio termodinámico es común idealizar los sistemas para que sus propiedades
mecánicas y eléctricas sean lo más triviales posibles. Cuando el contenido esencial de la
termodinámica se ha desarrollado, es una cuestión simple extender el análisis a sistemas con
estructuras mecánicas y eléctricas relativamente complejas. La cuestión esencial es señalar que
las restricciones en los tipos de sistemas considerados no son limitaciones básicas sobre la
generalidad de la teoría termodinámica, y sólo se adoptan meramente para la simplificación
expositiva.

Quizá la complicación principal del análisis termodinámico como herramienta en ingeniería se


deba a que es práctica común restringir los estudios a sistemas simples, definidos como sistemas
que son macroscópicamente homogéneos, isotrópicos, y desprovistos de carga eléctrica, que
son lo suficientemente grandes para que los efectos de frontera puedan ser ignorados, y que no
se encuentran bajo la acción de campos eléctricos, magnéticos o gravitacionales. Nada más
lejos de la realidad.

Básicamente la termodinámica en la ingeniería se enfoca en la producción de trabajo, a menudo


bajo la forma de energía cinética a partir de calor como resultado de los procesos de la
combustión, aunque no siempre es este el caso.

1
Dimensiones y Unidades
Las dimensiones fundamentales son primordiales, ya que son reconocidas por nuestras percepciones
sensoriales y no son definibles en términos de algo más simple. Sin embargo, su empleo requiere la
definición de escalas arbitrarias de medición, divididas en unidades de tamaño específico. Las unidades
originales fueron establecidas mediante un acuerdo internacional, y codificadas como el Sistema
Internacional de Unidades (abreviado como SI, para el Systéme International).

El segundo, la unidad del tiempo del SI, cuyo símbolo es s, corresponde a la duración de 9192631770
ciclos de radiación asociada a una transición específica del átomo del cesio. El metro, cuyo símbolo es
m, es la unidad fundamental de longitud que se define como la distancia que la luz viaja en el vacío
durante 1/299792458 de un segundo. El kilogramo, con el símbolo kg, es la masa de un cilindro de
platino/iridio que se conserva en la Oficina Internacional de Pesas y Medidas en Sevres, Francia. La
unidad de la temperatura es el kelvin, cuyo símbolo es K, y es igual a 1/273.16 de la temperatura
termodinámica del punto triple del agua. En las secciones posteriores se hace un análisis detallado de la
temperatura, que es la dimensión característica de la termodinámica. El mal, cuyo símbolo es mol, es la
cantidad de sustancia representada por tantas entidades elementales (por ejemplo, moléculas) como el
número de átomos que hay en 0.012 kg de carbono-12. Esto es equivalente al "gramo mol" usado de
manera común por los químicos. Los múltiplos y fracciones decimales de las unidades del SI se designan
mediante prefijos. Los de uso más frecuente se enumeran en la tabla 1.1. De esta manera, el centímetro
está determinado como 1cm = 10-2 m, y el kilogramo como 1kg = 103 g.

Otros sistemas de unidades, como el sistema inglés de ingeniería, utilizan unidades que están
relacionadas con las unidades del SI mediante factores fijos de conversión. Así, el pie se define como
0.3048 m, la libra masa (lbm) como 0.45359237 kg y la libra mol (lb mol) como 453.59237 mol.

Sistemas de unidades
Como se están considerando las propiedades termodinámicas desde un punto de vista
macroscópico, se manejan cantidades que pueden medirse o cuantificarse de manera directa o
indirecta. Lo anterior hace que las unidades sean relevantes en el estudio de la termodinámica.

Debido a que las cantidades físicas se relacionan entre sí por definiciones y leyes, solo es
necesario definir a un número relativamente pequeño de ellas para definir al resto. Las
cantidades físicas que se definen y a partir de las cuales se puede obtener el resto de ellas se
conocen como magnitudes primarias.

2
Las cantidades físicas que se definen a partir de las dimensiones primarias se conocen como
magnitudes secundarias.
Por ejemplo, la longitud y el tiempo son consideradas magnitudes primarias, mientras que la
velocidad (que puede definirse a partir de la longitud y el tiempo) es considerada secundaria.
Una vez adoptadas las dimensiones primarias es necesario definir las unidades base para cada
una de ellas. Las unidades del resto de las cantidades físicas (secundarias) serán definidas a
partir de las unidades base designadas para las dimensiones primarias.

Unidades básicas del SI y del USCS

Peso, masa, volumen, volumen específico y gravedad específica


Peso (W)
El peso de un sistema se relaciona con la fuerza gravitacional ejercida sobre dicho sistema. Su
magnitud se determina por la aceleración de la gravedad (g). En el ámbito terrestre toma un
valor en el Sistema Internacional (SI) de g = 9,806 m/s2 y en el sistema ingles de 32,174 ft/s2.
El peso de un sistema tomara valores diferentes según el marco de referencia, pues la fuerza
gravitacional depende del lugar donde se haga la medición. Por ejemplo, un sistema en la
superficie terrestre tiene un valor, pero si se toma como marco de referencia la Luna, el
sistema registrara un peso de 1/6 del valor registrado en la Tierra.

Masa (m)
Es una propiedad intrínseca de la materia y se relaciona con la cantidad de sustancia que
contiene un sistema termodinámico. Las unidades de la masa son el kilogramo (kg) en el SI.

Volumen (V)
Es otra propiedad intrínseca de la materia y se relaciona con las dimensiones en el espacio que
tendrá un sistema termodinámico. Las unidades del volumen son el metro cubico (m3) en el SI.

Densidad (𝝆)
Se define como la relación entre la masa y la unidad de volumen de un sistema termodinámico.
Sus dimensiones en el SI son el kg/m3:

Volumen específico (v)


Esta propiedad es el reciproco de la densidad y, por tanto, es la relación entre el volumen y la
masa de un sistema termodinámico. Sus unidades son el m3/kg en el SI:

3
El volumen especifico de un sistema en un campo gravitacional puede variar de un punto a otro. Un
ejemplo de ello se observa si se considera a la atmosfera como un sistema, donde puede notarse que el
volumen especifico crece conforme aumenta la altura.

Gravedad específica (GE) o densidad relativa


Se define comúnmente como la relación entre la densidad de una sustancia y la densidad del
agua a 4 °C.

Peso específico (γ)


Se define como la relación entre el peso por unidad de volumen. Las unidades del peso
específico en el Sistema Internacional son el Newton sobre metro cubico (N/m3):

Un aspecto fundamental en la termodinámica es la obtención de relaciones entre las propiedades


termodinámicas de un sistema que permiten observar la variación de una propiedad en función
de otras. Algunas de estas relaciones se basan en métodos empíricos, mientras que otras
provienen de un análisis matemático altamente riguroso. En general, siempre que se pueda
establecer una relación entre algunas de las propiedades de un sistema se dice que se ha obtenido
una ecuación de estado.

Matemáticamente se puede decir que una ecuación de estado es la siguiente:


F(x1, x2, x3…, xi)
donde x representa una propiedad de un sistema y el subíndice se asocia con la i-esima
propiedad.

Ejemplo 1

4
Presión(P)
Cuando se habla del efecto de una fuerza aplicada en dirección normal a un sólido, se hace
referencia a esfuerzos. Sin embargo, para líquidos y gases se trata de presión.
La presión en un fluido en reposo en cualquier punto es la misma en todas direcciones y se
define como la componente normal de la fuerza por unidad de área

La presión (símbolo p) es una magnitud física que mide la proyección de la fuerza en dirección
perpendicular por unidad de superficie, y sirve para caracterizar cómo se aplica una determinada
fuerza resultante sobre una línea. En el Sistema Internacional de Unidades la presión se mide
en una unidad derivada que se denomina pascal (Pa) que es equivalente a una fuerza total de un
newton (N) actuando uniformemente en un metro cuadrado (m²). En el Sistema Inglés la presión
se mide en libra por pulgada cuadrada (pound per square inch o psi) que es equivalente a una
fuerza total de una libra actuando en una pulgada cuadrada.

La presión es la magnitud escalar que relaciona la fuerza con la superficie sobre la cual actúa,
es decir, equivale a la fuerza que actúa sobre la superficie. Cuando sobre una superficie plana
de área A se aplica una fuerza normal F de manera uniforme, la presión P viene dada de la
siguiente forma:
𝐅
𝐏=
𝐀
Donde
𝑃 = 𝑃𝑟𝑒𝑠𝑖𝑜𝑛
𝐹 = 𝐹𝑢𝑒𝑟𝑧𝑎
𝐴 = 𝐴𝑟𝑒𝑎 𝑡𝑜𝑡𝑎𝑙 𝑑𝑒 𝑙𝑎 𝑠𝑢𝑝𝑒𝑟𝑓𝑖𝑐𝑖𝑒

La presión P en un punto de un fluido en equilibrio es la misma en todas direcciones, y la


relación anterior puede expresarse de manera más simple como:

Unidades de medida, presión y sus factores de conversión


La presión atmosférica media es de 101 325 pascales (101,3 kPa), a nivel del mar, donde 1 atm
= 1,01325 bar = 101325 Pa = 1,033 kgf/cm² y 1 m.c.a = 9,81 kPa.

5
La presión P ejercida por un fluido sobre una superficie se define como la fuerza normal
ejercida por el fluido por unidad de área de la superficie. Si la fuerza se mide en N el área en la
𝑁
unidad es el newton por metro cuadrado o 𝑚2 , llamado pascal representado con el símbolo Pa,
es la unidad básica de la presión para el SI. En el sistema inglés de ingeniería una unidad común
es la libra fuerza por pulgada cuadrada (psi, por sus siglas en inglés).
El estándar primario para mediciones de presión es la balanza de peso muerto, en la cual una
fuerza conocida se equilibra con una presión del fluido que actúa sobre un área conocida, donde
P == F/A. En la figura 1.1 se muestra un diseño sencillo. El pistón se ajusta cuidadosamente al
cilindro dejando una separación muy pequeña. Se ponen pesas en la bandeja hasta que la presión
del aceite, que tiende a hacer que el pistón se eleve, se equilibre precisamente por la fuerza de
gravedad en el pistón y todo lo que éste soporta. Con esta fuerza dada por la ley de Newton la
presión del aceite es:
𝐅 𝒎𝒈
𝐏= =
𝐀 𝑨
Donde m es la masa del pistón, de la bandeja y de las pesas; g es la aceleración local de la
gravedad, y A es el área de sección transversal del pistón. Los instrumentos de medida de uso
común, como el manómetro de Bourdon, se calibran por comparación con las balanzas de peso
muerto.

Figura 1.2: Balanza de peso muerto.

Puesto que una columna vertical de un fluido determinado que está bajo la influencia de la
gravedad, ejerce una presión en su base que va en proporción directa con su altura, la presión
también se expresa como la altura equivalente de una columna de fluido. Éste es el fundamento
para el uso de manómetros en mediciones de presión. La conversión de la altura a fuerza por
unidad de área surge a partir de la ley de Newton aplicada a la fuerza de la gravedad que actúa
sobre la masa del fluido en la columna. La masa está dada por: 𝑚 = 𝐴ℎ𝜌 donde A es el área de
sección transversal de la columna, h es su altura y 𝜌 es la densidad del fluido. Por lo tanto,

𝐅 𝒎𝒈 𝑨𝒉𝝆𝒈
𝐏= = = = 𝒉𝝆𝒈
𝐀 𝑨 𝑨
La presión que corresponde a una altura del fluido se determina por la densidad del mismo (que
depende de su identidad y temperatura) y de la aceleración local de la gravedad. Así, el (torr)

6
es la presión equivalente de 1 milímetro de mercurio a 0 °C en un campo gravitacional estándar,
y es igual a 133.322 Pa.
Otra unidad de presión es la atmósfera están dar (atm), que es la presión promedio ejercida por
la atmósfera de la Tierra a nivel del mar, y se define como 101 325 Pa, 101.325 kPa, o bien,
0.101325 MPa. El bar, una unidad del SI, se define como 105 Pa y es igual a 0.986923 (atm).
La mayoría de los manómetros de presión dan lecturas que representan la diferencia entre la
presión de interés y la presión atmosférica de los alrededores. Estas lecturas se conocen como
presiones mano métricas y se pueden convertir a presiones absolutas al sumarles la presión
barométrica. En los cálculos termodinámicos se deben utilizar presiones absolutas.

Las unidades correspondientes en el SI son el N/m2 o Pascal (Pa). Con frecuencia en el estudio
de la termodinámica se encuentran múltiplos de la unidad de presión, como el kilopascal (1 kPa
= 1 . 103 Pa) o el megapascal (1 MPa = 1 . 106 Pa). Asimismo, se encuentran sus equivalencias
en otros sistemas de unidades en donde se utilizan el bar, la atmosfera (atm), el kilogramo por
metro cuadrado (kg/m2) y la psi (lbf/in2) para el sistema inglés. Las equivalencias entre uno y
otro sistema se dan a continuación:

Ejemplo 2
Se utiliza una balanza de peso muerto con un pistón de 1 cm de diámetro para medir presiones
con mucha precisión. En un caso particular, una masa de 6.14 kg (incluyendo el pistón y la
bandeja) alcanza el equilibrio. Si la aceleración local de la gravedad es 9.82 m/s2, ¿cuál es la
presión manométrica medida? Si la presión barométrica es 748(torr), ¿cuál es la presión
absoluta?

Solución 2
La fuerza ejercida por la gravedad sobre el pistón, la bandeja las pesas es:
F = mg = (6.14) (9.82) = 60.295 N

𝐹 60.295
Presión manométrica = = (1/4)(𝜋)(1)2
= 76.77 N/cm2
𝐴
Por lo tanto la presión absoluta es:
F= 76.77 + (748) (0.013332) = 86.74 N/cm2
o
P = 867.4 kPa

Ejemplo 3
A 27°C la lectura en un manómetro lleno de mercurio es de 60.5 cm. La aceleración local de la
gravedad es 9.784 m/s2. ¿A qué presión le corresponde esta altura del mercurio?

Solución 3
Recuerde la ecuación del texto anterior, 𝑃 = ℎ𝜌𝑔. A 27 °C la densidad del mercurio es
13.53 g/cm3. Por esto,

7
P= 60.5 cm x 13.53 g/cm3 x 9.784 m/s2 = 8009 (g m)/(s2 cm2)
o
P= 8.009 (kg m)/(s2 cm2) = 8.009 N/cm2 = 80.09 kPa = 0.8009 bar

Las obsoletas unidades manométricas de presión, como los milímetros de mercurio, están
basadas en la presión ejercida por el peso de algún fluido de referencia bajo cierta gravedad
estándar. También se utilizan los milímetros de columna de agua.

Temperatura(T)
La temperatura se mide por lo regular con termómetros de líquidos en capilares de vidrio, donde
el fluido se expande cuando se calienta. De este modo, un tubo uniforme que está parcialmente
lleno de mercurio, alcohol o algún otro fluido, indica el grado de "calentamiento" mediante la
longitud de la columna del fluido. De cualquier modo, se asignan valores numéricos a los
diversos grados de calentamiento por medio de una definición arbitraria.

Para la escala Celsius el punto de hielo (punto de congelación del agua saturada con aire a la
presión atmosférica estándar) es cero, y el punto de vapor (punto de ebullición del agua pura a
la presión atmosférica estándar) es 100. Es posible asignar a un termómetro una escala numérica
sumergiéndolo en un baño de hielo y haciendo una marca correspondiente al cero en el nivel
donde se encuentra el fluido, para después sumergido en agua hirviendo y hacer una marca para
el valor de 100 en este nuevo nivel del fluido. La distancia entre las dos marcas se divide en
100 espacios equidistantes denominados grados. Para extender la escala del termómetro se
marcan otros espacios de igual tamaño por debajo del cero y por encima del 100.

Todos los termómetros, sin importar el fluido que contengan, proporcionan la misma lectura en
cero y en 100 si se calibran con este método, pero generalmente en otros puntos no
corresponden las lecturas porque los fluidos varían en sus características de expansión. De esta
manera se requiere una elección arbitraria del fluido, y la escala de temperatura en el sistema
SI cuya unidad es el kelvin y su símbolo es K, se apoya en el gas ideal como fluido
termométrico. Puesto que la definición de la escala Kelvin depende de las propiedades de los
gases, se hará un análisis detallado hasta el capítulo que corresponda. Sin embargo, se observa
que la escala absoluta depende del concepto de un límite inferior de la temperatura.

Las temperaturas Kelvin se indican con el símbolo T, en tanto que las temperaturas Celsius se
designan con el símbolo t, y se definen en relación a las temperaturas Kelvin mediante:
t °C = TK - 273.15

La unidad de temperatura Celsius es el grado Celsius, °C, que en magnitud equivale al Kelvin.
No obstante, las temperaturas en la escala Celsius son 273.15 grados menores que en la escala
8
Kelvin. De esta forma el límite inferior de la temperatura, conocido como el cero absoluto en
la escala Kelvin, se presenta en - 273.15℃.

En la práctica, la Escala Internacional de Temperatura de 1990 (ITS-90) se utiliza para la


calibración de instrumentos científicos e industriales. La escala ITS-90 se define de modo que
sus valores difieren de las temperaturas del gas ideal, pero que están en los límites de exactitud
con los que actualmente se hacen las mediciones. Se basa en valores asignados de la temperatura
para un número de estados de equilibrio de fase reproducibles de sustancias puras (puntos fijos)
y en los instrumentos estándar calibrados a estas temperaturas. La interpolación entre las
temperaturas de punto fijo se proporciona mediante fórmulas que establecen la relación entre
las lecturas de los instrumentos estándar y los valores de la ITS-90. El termómetro de resistencia
de platino es un ejemplo de un instrumento estándar, que se utiliza para temperaturas que van
desde -259.35 °C (punto triple del hidrógeno) hasta 961.78 °C (el punto de congelación de la
plata).

Además de las escalas Kelvin y Celsius existen otras dos que aún son utilizadas por los
ingenieros de Estados Unidos: las escalas Rankine y Fahrenheit.

La escala en grados Celsius está relacionada con la escala absoluta en Kelvin por la siguiente
relación:

La relación entre la escala absoluta en el SI y su análoga en el sistema ingles está dada por la siguiente
ecuación:

La equivalencia entre la escala absoluta Rankine y la escala en grados Fahrenheit es:

Finalmente, la relación entre las escalas entre el SI y el sistema inglés es:

La escala Fahrenheit se relaciona con la de Rankine por una ecuación semejante a la relación
entre las escalas Celsius Kelvin:

t(℉) = T(R) - 459.67

De este modo, el límite inferior de la temperatura en la escala Fahrenheit es -459.67(℉).

Por consiguiente, el punto de congelación del agua es 32(℉) y el punto de ebullición normal
del agua es 212(℉).

El grado Celsius y el kelvin representan el mismo intervalo de temperatura, al igual que el grado
Fahrenheit y el Rankine. Las relaciones entre las cuatro escalas de temperatura se ilustran en la
figura 1.2. En termodinámica, se da a entender una temperatura absoluta mediante una
referencia informal de la temperatura.

9
Figura 1.2: Relaciones entre las escalas de temperatura.

En la figura 1.3 se aprecia una comparación entre las diferentes escalas de temperatura.

Figura 1.3 Escala de temperatura.

Es importante mencionar que cuando se trata con diferencias de temperatura ΔT (gradientes de


temperatura o deltas), los incrementos son los mismos en las escalas absolutas que en sus
respectivas escalas comunes, es decir:
ΔT(K) = ΔT(°C)
10
ΔT(R) = ΔT(°F)

Temperatura Celsius
En la práctica, las temperaturas se expresan en la escala Celsius, en la cual el punto cero se
consideran como el punto de congelación del agua, definido como la temperatura de equilibrio
del hielo y aire saturado de vapor de agua a la presión de 1 atmósfera (atm).

En forma experimental, se encuentra que el punto de congelación del agua es 0.01 K inferior al
punto triple del agua, y por lo tanto, es 273.15 K. La temperatura Celsius (°C) se define por:

Fórmulas de conversión de grados Celsius, Fahrenheit, Kelvin, Rankine y


Réaumur.
De Fahrenheit a Celsius

De Celsius a Fahrenheit

De Kelvin a Celsius

De Celsius a Kelvin

De Kelvin a Fahrenheit

De Fahrenheit a Kelvin

De Rankine a Fahrenheit

De Fahrenheit a Rankine

11
De Réaumur a Celsius

De Rankine a Kelvin

De Rankine a Celsius

De Celsius a Rankine

De Celsius a Réaumur

De Kelvin a Rankine

De Fahrenheit a Réaumur

De Réaumur a Fahrenheit

De Kelvin a Réaumur

De Réaumur a Kelvin

De Rankine a Réaumur

De Réaumur a Rankine

Ejemplo 1.4

12
En la escala Celsius, la temperatura del vapor de agua medida en forma experimental, que
corresponde al punto de ebullición del agua a la presión de 1 atm, es 100.00 °C.
Inicialmente, la temperatura de una habitación es de 20 °C. Después de un tiempo, debido a
factores ambientales, la temperatura aumenta a 25 °C. Expresa las temperaturas inicial y final
en °F, K y R. Además, especifica las diferencias de temperatura en °C, °F, K y R.
Solución
Convirtiendo las temperaturas a grados Fahrenheit:
Para 20 °C 1,8(20 °C) + 32 = 68 °F
Para 25 °C 1,8(25 °C) + 32 = 77 °F

Convirtiendo las temperaturas a Kelvin:


Para 20 °C 20 °C +273,15 = 293,15 K
Para 25°C 25 °C +273,15 = 298,15 K

Convirtiendo las temperaturas a Rankine:


Para 20 °C 1,8(293,15 K) = 527,67 R, o bien, 68 °F + 459,67 = 527,67 R
Para 25 °C 1,8(298,15 K) = 536,67 R, o bien, 77 °F + 459,67 = 536,67 R

Obteniendo las diferencias de temperatura (ΔT = Tfinal − Tinicial):


Para grados Celsius:
ΔT = 25 °C − 20 °C = 5 °C
Para Kelvin:
ΔT = 298,15 K − 293,15 K = 5 K
Para grados Fahrenheit:
ΔT = 77 °F − 68 °F = 9 °C
Para Rankine:
ΔT = 536,67 R − 527,67 R = 9 R
Puede observarse que la diferencia de temperaturas tiene el mismo valor para la escala Celsius
y la escala Kelvin. De igual manera, dicha diferencia tiene el mismo valor para la escala
Fahrenheit y para la escala Rankine.

Ecuaciones básicas
Las proporcionalidades básicas, cada una escrita como una ecuación con su propio factor de
proporcionalidad, son

Trabajo: Trabajo (física): Es el producto de una fuerza aplicada sobre un cuerpo y del
desplazamiento del cuerpo en la dirección de esta fuerza. Mientras se realiza trabajo sobre el
cuerpo, se produce una transferencia de energía al mismo, por lo que puede decirse que el
trabajo es energía en movimiento.

Si una fuerza aplicada → está en la misma dirección que el desplazamiento ∆ →, por lo


𝐹 𝑟
tanto 𝜃 = 0 y cos 𝜃 ≠ 1. En este caso, la ecuación 1 produce

13
𝑾 = 𝑭 𝐝 𝐜𝐨𝐬 𝜽
Las unidades de trabajo son las de fuerza multiplicada por longitud. En consecuencia, la unidad
del SI de trabajo es el newton·metro (𝑁 . 𝑀 = 𝑘𝑔. 𝑚2 /𝑠 2 ). Esta combinación de unidades se
usa con tanta frecuencia que se le ha dado un nombre propio, joule (J).

Ejemplo 1.5
Un hombre que limpia un piso jala una aspiradora con una fuerza de magnitud F = 50.0 N en
un ángulo de 30.0° con la horizontal (figura 7.5). Calcule el trabajo consumido por la fuerza
sobre la aspiradora a medida que esta se desplaza 3.00 m hacia la derecha.
Solución
Conceptualizar La figura 1.4 ayuda a formar ideas de la situación. Piense en una experiencia
de su vida en la que jalo un objeto a través del piso con una soga o cuerda.
Categorizar Se aplica una fuerza sobre un objeto, un desplazamiento del objeto y el ángulo
entre los dos vectores, de modo que este ejemplo se clasifica como un problema de sustitución.
La aspiradora se identifica como el sistema.
Aplique la definición de trabajo :

Figura 1.4 (Ejemplo 7.1) Una aspiradora se jala con un ángulo de 30.0° de la horizontal.

Fuerza: En el campo de la física, la fuerza es una magnitud vectorial, y es toda causa capaz
de cambiar el estado de reposo o de movimiento de un cuerpo. La fuerza que actúa sobre un
objeto de masa m es igual a la variación del momento lineal (o cantidad de movimiento) de
dicho objeto respecto del tiempo. La unidad de fuerza en el Sistema Internacional (SI) es el
newton, de símbolo N. El concepto de fuerza se suele explicar matemáticamente en términos
de las tres leyes del movimiento de Newton.

En una fuerza pueden tenerse en cuenta diferentes rasgos determinantes: el punto de aplicación
(punto del cuerpo sobre el que se ejerce la fuerza); la dirección (recta sobre la que la fuerza
induce a moverse al cuerpo); el sentido (orientación de la fuerza) y la intensidad (medida de la
fuerza respecto a una unitaria establecida).

Existen dos tipos de fuerzas; las que actúan por contacto, en donde el cuerpo que ejerce la fuerza
está en contacto directo con el cuerpo sobre el que esta se aplica, por ejemplo: lanzar una piedra,
tirar de una cuerda, etc. Y las que actúan a distancia, aquí el cuerpo el cuerpo que ejerce la

14
fuerza no está en contacto con el cuerpo sobre el que esta se aplica, ejemplo: la fuerza de
atracción magnética, la fuerza con que la Tierra atrae a los cuerpos, etc.

Segunda Ley de Newton


La segunda ley de Newton como se establece más abajo, se aplica en un gran número de
fenómenos físicos, pero no es un principio fundamental como lo son las leyes de conservación.
Aplica solamente si la fuerza es una fuerza neta externa. No aplica directamente en situaciones
donde la masa cambia, ya sea perdiendo o ganando material o si el objeto está viajando cerca
de la velocidad de la luz, en cuyo caso deben incluirse los efectos relativistas. Tampoco aplica
en escalas muy pequeñas a nivel del átomo, donde debe usarse la mecánica cuántica.

Pruebe a entrar datos en las casillas de abajo. Especificando dos cantidades cualesquiera, puede
obtenerse la tercera. Después de introducir los dos valores, pulse sobre la casilla vacía para
obtener su valor.

La Segunda Ley de Newton establece lo siguiente:

La aceleración de un objeto es directamente proporcional a la fuerza neta que actúa sobre él e


inversamente proporcional a su masa.
𝑭=𝑴∗𝒂
De esta forma podemos relacionar la fuerza y la masa de un objeto con el siguiente enunciado:
Una buena explicación para misma es que establece que siempre que un cuerpo ejerce una
fuerza sobre un segundo cuerpo, el segundo cuerpo ejerce una fuerza sobre el primero cuya
magnitud es igual, pero en dirección contraria a la primera. También podemos decir que la
segunda ley de Newton responde la pregunta de lo que le sucede a un objeto que tiene una
fuerza resultante diferente de cero actuando sobre él.

La siguiente ecuación es la ley de Newton de la gravitación, que expresa la fuerza de atracción


entre dos partículas de masas ma y mb separadas entre sí una distancia r.

𝒎𝒂 ∗ 𝒎𝒃
𝑭=
𝒓𝟐

Ejemplo 1.6. Una fuerza le proporciona a la masa de 2,5 Kg, una aceleración de 1,2 m/s 2.
Calcular la magnitud de dicha fuerza en Newton y dinas.
Datos
m = 2,5 Kg.
a =1,2 m/s2.
F =? (N y dinas)
Solución
Nótese que los datos aparecen en un mismo sistema de unidades (M.K.S.)
Para calcular la fuerza usamos la ecuación de la segunda ley de Newton:
𝐹 =𝑀∗𝑎
Sustituyendo valores tenemos:
𝑚 𝑚
𝐹 = 2,5 𝐾𝑔 ∗ 1,2 2 = 3 𝐾𝑔 ∗ 2 = 3 𝑁
𝑠 𝑠
Como nos piden que lo expresemos en dinas, bastará con multiplicar por 105, luego:
105 𝑑𝑖𝑛𝑎𝑠
𝐹 =3𝑁 = 3𝑁 ( ) = 3 ∗ 105 𝑑𝑖𝑛𝑎𝑠
1𝑁
15
Ejemplo 1.7. ¿Qué aceleración adquirirá un cuerpo de 0,5 Kg. cuando sobre él actúa una fuerza
de 200000 dinas?
Datos
a =?
m = 2,5 Kg.
F = 200000 dinas
Solución
La masa está dada en M.K.S., en cambio la fuerza está dada en c.g.s.
Para trabajar con M.K.S. debemos transformar la fuerza a la unida M.K.S. de esa magnitud

1𝑁
2 ∗ 105 𝑑𝑖𝑛𝑎𝑠 ( 5 )= 2𝑁
10 𝑑𝑖𝑛𝑎𝑠

La ecuación de la segunda ley de Newton viene dada por:


𝐹 =𝑀∗𝑎
Despejando a tenemos:
𝐹
𝑎=
𝑚
Sustituyendo sus valores se tiene:
𝑚
2𝑁 𝐾𝑔 2 𝑚
𝑎= =4 𝑠 =4 2
0.5 𝐾𝑔 𝐾𝑔 𝑠
𝑚
𝑎=4 2
𝑠

Ejemplo 1.8. Un ascensor pesa 400 Kp. ¿Qué fuerza debe ejercer el cable hacia arriba para que
suba con una aceleración de 5 m/s2? Suponiendo nulo el roce y la masa del ascensor es de 400
Kg.
Solución
Sobre el ascensor actúan dos fuerzas: la fuerza F de tracción del cable y la fuerza P del peso,
dirigida hacia abajo.

La fuerza resultante que actúa sobre el ascensor es F – P


Aplicando la ecuación de la segunda ley de Newton tenemos:
F – P = m.a

Al transformar 400 Kp a N nos queda que:


400 Kp = 400 (9,8 N) = 3920 N
Sustituyendo los valores de P, m y a se tiene:
F – 3920 N = 400 Kg.(0,5 m/s2)
F – 3920 N = 200 N
Si despejamos F tenemos:
F = 200 N + 3920 N
F = 4120 N

El kilopondio (de kilo- y el latín pondus, -ĕris 'peso') o kilogramo-fuerza es la unidad


de fuerza en el antiguo Sistema Técnico de Unidades.1 Es una de las tres unidades
fundamentales de este sistema; las otras dos son el metro (longitud) y el segundo (tiempo).

Un kilopondio o kilogramo-fuerza, es la fuerza ejercida sobre una masa de 1 kg masa (según se


define en el SI) por la gravedad estándar en la superficie terrestre, esto es 9,80665 m/s2. En
16
definitiva, el kilogramo-fuerza (o kilopondio) es lo que pesa una masa de 1 kg en la superficie
terrestre.

El término kilopondio es escasamente utilizado, tanto en el ámbito científico y técnico como en


la práctica cotidiana. Normalmente no oiremos decir "yo peso 70 kilopondios o kilogramos-
fuerza" (que sería lo correcto si utilizamos el Sistema Técnico de Unidades), o "yo peso 686 N",
o "yo tengo una masa de 70 kilogramos" (si utilizamos el SI), sino que lo común es decir: "yo
peso 70 kilogramos o kilos" (donde kilogramo es la unidad de masa del SI), a pesar de que, en
realidad, nos estamos refiriendo a kilogramos-fuerza, y no a kilogramos de masa. En lo anterior,
debemos interpretar a la expresión "kilos" como acortamiento coloquial de kilogramos-fuerza
o kilopondios, ya que estamos hablando de un peso; es decir, de una fuerza y no de una masa.

El valor estándar de la gravedad (g) terrestre es de 9,80665 m/s². Entonces (y de acuerdo con la
segunda ley de Newton: fuerza = masa × aceleración), tendremos:
1kp = 1 kgf = 1 kg * 9,80665 m/s2 = 9,80665 N
de modo que 1 kilogramo-fuerza o kilopondio equivale a 9,80665 N según la constante
gravitacional de la tierra. Por lo que según la aceleración de la gravedad y las unidades, el kgf =
kp = kg.
Según las constantes gravitacionales de cada planeta, para pasar de kp a kg, tendríamos que
dividir los kilopondios/9,80665 y multiplicar por la constante gravitacional del planeta a
estudiar.

El kilogramo-fuerza o kilopondio (SI) representa el peso de una masa de 1 kg (SI) en la


superficie terrestre. Esta circunstancia ha dado lugar a cierto desconcierto que parte de la
confusión inicial entre los conceptos de peso y masa.
Destaquemos un ejemplo: en la Luna ese mismo kg de masa va a pesar solamente 0,1666 kp o
kgf (o 1,634 N si usamos el SI, ya que la gravedad lunar es la sexta parte de la gravedad
terrestre).
Resumiendo
 1 kg masa (SI) es igual a 0,102 utm (STU).
 Además, el kg de masa pesa:
 en la Tierra: 1 kilopondio o kilogramo-fuerza (STU), o 9,80665 N (SI).
 en la Luna: 0,1666 kilopondios o kilogramos-fuerza (STU), o 1,634 N.
Sin embargo, su masa permanecerá invariable: 1 kg masa (SI) o 0,102 utm, tanto en la Tierra
como en la Luna u otro lugar.

Ejemplo 1.9. Un carrito con su carga tiene una masa de 25 Kg. Cuando sobre él actúa,
horizontalmente, una fuerza de 80 N adquiere una aceleración de 0,5 m/s2. ¿Qué magnitud tiene
la fuerza de rozamiento Fr que se opone al avance del carrito?

Solución
La fuerza F, que actúa hacia la derecha, es contrarrestada por la fuerza de roce Fr, que actúa
hacia la izquierda. De esta forma se obtiene una resultante F – Fr que es la fuerza que produce
el movimiento.
Si aplicamos la segunda ley de Newton se tiene:
Sustituyendo F, m y a por sus valores nos queda
80 N – Fr = 25 Kg. (0,5 m/s2)
80 N – Fr = 12,5 N
Si despejamos Fr nos queda:
Fr = 80 N – 12,5 N
Fr = 67,5 N

17
Ejemplo 1.10. ¿Cuál es la fuerza necesaria para que un móvil de 1500 Kg., partiendo de
reposo adquiera una rapidez de 2 m/s2 en 12 s?
Datos
F =?
m = 1500 Kg.
Vo = 0
Vf = 2 m/s2
t = 12 s
Solución
Como las unidades están todas en el sistema M.K.S. no necesitamos hacer transformaciones.
La fuerza que nos piden la obtenemos de la ecuación de la segunda ley de Newton: F = m.a
De esa ecuación conocemos la masa, pero desconocemos la aceleración. Esta podemos
obtenerla a través de la ecuación
𝑉𝑓
𝑎=
𝑡
Porque partió de reposo.
Sustituyendo Vf y t por sus valores tenemos:

2 𝑚/𝑠 𝑚
𝑎= = 0.16
12 𝑠 𝑠2

Si sustituimos el valor de a y de m en la ecuación (I) tenemos que:


𝑚
𝐹 = 1500 𝐾𝑔 ∗ 0.16 2
𝑠
𝐹 = 240 𝑁

Ejemplo 1.11. Calcular la masa de un cuerpo, que estando de reposo se le aplica una fuerza
de 150 N durante 30 s, permitiéndole recorrer 10 m. ¿Qué rapidez tendrá al cabo de ese
tiempo?
Datos
m =?
Vo = 0
F = 150 N
t = 30 s
x = 10 m
Vf =?
Solución
Como nos piden la masa, despejamos la segunda la segunda ley de Newton:

𝐹 =𝑚∗𝑎 (I)

Como no se conoce la aceleración y nos dan la distancia que recorre partiendo de reposo,
usamos la ecuación de la distancia en función del tiempo y despejamos (a)
Sustituyendo valores tenemos:
2𝑋
𝑎= (II)
𝑡2
𝑎∗𝑡 2
𝑋= De donde 2𝑋 = 𝑎 ∗ 𝑡 2
2

Sustituyendo los valores de X y t en (II) tenemos:

18
2∗10 𝑚 20 𝑚 𝑚
𝑎= = = 0.02
(30 𝑠)2 900 𝑠 2 𝑠2
𝑚
𝑎 = 0.02
𝑠2
Sustituyendo a y F por sus valores en (I):
150 𝑁 𝐾𝑔. 𝑚/𝑠 2
𝑚= = 7500
0.02 𝑚/𝑠 2 𝑚/𝑠 2
𝑚 = 7500 𝐾𝑔

Ejemplo 1.12. Un astronauta pesa 730 N en Houston, Texas, donde la aceleración de la


gravedad local es de g = 9.792 m/s2 ¿Cuáles son la masa y el peso del astronauta en la Luna,
donde g = 1.67 m/s2?
Solución
Con a =g, la ley de Newton es: F =mg, De donde,

𝐹 730 𝑁 𝑁 𝑠2
𝑚= = = 74.55
𝑔 9.792 𝑚/𝑠 2 𝑚

𝑘𝑔 𝑚
Como el newton N tiene unidades de 2 ,
𝑠
m = 74.55 kg

Esta masa del astronauta es independiente de la ubicación, pero el peso depende de la


aceleración local de la gravedad. De esta forma, el peso del astronauta en la Luna es:

F(Luna) = mg(Luna) = 74.55 kg x 1.67 m/s2


F(Luna) = 124.5 kg m/s2 = 124.5 N

El uso del sistema inglés de unidades de ingeniería requiere de la conversión del peso del
astronauta a (lbf) y de los valores de g a (pie/s2). Puesto que 1 N equivale a 0.224809 (lbf) y 1
m a 3.28084 (pie):

El peso del astronauta en Houston = 164.1 (lbf)

g(Houston) = 32.13 y g(Luna) = 5.48(pie/s2)

Por esto, la ley de Newton da:


𝐹𝑔𝑐 164.1 (𝑙𝑏𝑓 )𝑥32.1740 (𝑙𝑏𝑚 )(𝑓𝑡)(𝑙𝑏𝑓 )−1 (𝑠)−2
𝑚= =
𝑔 32.13 (𝑓𝑡)(𝑠)−2
o m = 164.3(lbm)

De esta forma, la masa del astronauta en (lbm) Y su peso en (lbf) en Houston son casi
numéricamente iguales, pero en la Luna éste no es el caso:

𝑚𝑔(𝐿𝑢𝑛𝑎) (164.3)(5.48)
F(Luna) = = = 28.0(lbf )
𝑔𝑐 32.1740

Potencia: La potencia es la cantidad de trabajo que se realiza por unidad de tiempo. Puede
asociarse a la velocidad de un cambio de energía dentro de un sistema, o al tiempo que demora
la concreción de un trabajo. Se puede indicar que la potencia es la fuerza, el poder o la capacidad
para conseguir algo.
19
Nos indica la rapidez con la que una fuerza realiza un trabajo.
En el S.I. la unidad de potencia es el vatio(W).

En física, potencia (símbolo P) es la cantidad de trabajo efectuado por unidad de tiempo.

Como w = F*d, P = F · d / t, si la fuerza se desplaza a velocidad constante,


d/t = v, entonces :
P=F·v

Energía potencial
La energía potencial es la energía mecánica asociada a la localización de un cuerpo dentro de
un campo de fuerzas (gravitatoria, electrostática, etc.) o a existencia de un campo de fuerzas en
el interior de un cuerpo (energía elástica). La energía potencial de un cuerpo es una
consecuencia de la que el sistema de fuerzas que actúa sobre un cuerpo sea conservativo.

Independientemente de la fuerza que la origine, la energía potencial que posee el sistema físico
representa la energía "almacenada" en virtud de su posición y/o configuración, por
contraposición con la energía cinética que tiene y que representa su energía debida al
movimiento. Para un sistema conservativo, la suma de energía cinética y potencial es constante,
eso justifica el nombre de fuerzas conservativas, es decir, aquellas que hacen que la energía "se
conserve". El concepto de energía potencial también puede usarse para sistemas físicos en los
que intervienen fuerzas disipativas, y que por tanto no conservan la energía, sólo que en ese
caso la energía mecánica total no será constante, y para aplicar el principio de conservación de
la energía es necesario contabilizar la disipación de energía.

El valor de la energía potencial depende siempre del punto o configuración de referencia


escogido para medirla, por esa razón se dice a veces que físicamente sólo importa la variación
de energía potencial entre dos configuraciones.

La energía potencial interviene como se ha mencionado en el principio de conservación de la


energía y su campo de aplicación es muy general. Está presente no solo en la física clásica, sino
también de la física relativista y física cuántica. El concepto se ha generalizado también a la
física de partículas, donde se han llegado a utilizar potenciales complejos con el objeto de
incluir también la energía disipada por el sistema.

La Energía Potencial es la capacidad que tiene un cuerpo para realizar un trabajo de acuerdo a
la configuración que ostente en el sistema de cuerpos que ejercen fuerzas entre sí, es decir, la
energía potencial es la energía que es capaz de generar un trabajo como consecuencia de la
posición de un cuerpo.

Se expresa por la siguiente ecuación:

20
𝑬𝒑 = 𝒎𝒈𝒉

Donde:
𝐸𝑝 = 𝐸𝑛𝑒𝑟𝑔𝑖𝑎 𝑝𝑜𝑡𝑒𝑛𝑐𝑖𝑎𝑙
𝑚 = 𝑚𝑎𝑠𝑎 𝑑𝑒𝑙 𝑐𝑢𝑒𝑟𝑝𝑜
𝑔 = 𝑎𝑐𝑒𝑙𝑒𝑟𝑎𝑐𝑖𝑜𝑛 𝑑𝑒 𝑙𝑎 𝑔𝑟𝑎𝑣𝑒𝑑𝑎𝑑
ℎ = 𝑎𝑙𝑡𝑢𝑟𝑎 𝑑𝑒 𝑒𝑙𝑒𝑣𝑎𝑐𝑖𝑜𝑛 𝑑𝑒𝑙 𝑐𝑢𝑒𝑟𝑝𝑜

𝑬𝒑 = 𝒎𝒈(𝒉𝟐 − 𝒉𝟏 )

Energía cinética
En física, la energía cinética de un cuerpo es aquella energía que posee debido a su movimiento.
Se define como el trabajo necesario para acelerar un cuerpo de una masa determinada desde el
reposo hasta la velocidad indicada. Una vez conseguida esta energía durante la aceleración, el
cuerpo mantiene su energía cinética salvo que cambie su velocidad. Para que el cuerpo regrese
a su estado de reposo se requiere un trabajo negativo de la misma magnitud que su energía
cinética. Suele abreviarse con letra Ec.
Se expresa en Joule (J). 1 J = 1 kg·m2/s2. Estos son descritos por la velocidad de la masa
puntual, así:
𝒎𝒗𝟐
𝑬𝒄 =
𝟐
Donde
𝐸𝑐 = 𝐸𝑛𝑒𝑟𝑔𝑖𝑎 𝑐𝑖𝑛𝑒𝑡𝑖𝑐𝑎
𝑚 = 𝑚𝑎𝑠𝑎 𝑑𝑒𝑙 𝑐𝑢𝑒𝑟𝑝𝑜
𝑣 = 𝑣𝑒𝑙𝑜𝑐𝑖𝑑𝑎𝑑

𝟏
𝑬𝒄 = 𝒎(𝒗𝟐𝒇 − 𝒗𝟐𝒊 )
𝟐

Donde
𝑣𝑓2 : 𝑉𝑒𝑙𝑜𝑐𝑖𝑑𝑎𝑑 𝑓𝑖𝑛𝑎𝑙 𝑑𝑒𝑙 𝑐𝑢𝑒𝑟𝑝𝑜
𝑣𝑖2 : 𝑉𝑒𝑙𝑜𝑐𝑖𝑑𝑎𝑑 𝑖𝑛𝑖𝑐𝑖𝑎𝑙 𝑑𝑒𝑙 𝑐𝑢𝑒𝑟𝑝𝑜
𝐸𝑐 : Es la energía cinética del cuerpo en movimiento. Su unidad de medida en el Sistema
Internacional es el Julio (J)
m: Masa del cuerpo en movimiento. Su unidad de medida en el Sistema Internacional es el
Kilogramo (Kg)
v: Valor de la velocidad del cuerpo en movimiento. Su unidad de medida en el Sistema
Internacional es el metro por segundo (m/s)

21
Problema No 1.13: Calcula la energía cinética traslacional en Joule que lleva una bala de 8 g si
la magnitud de su velocidad es 400 m/s
Datos:
Ec = ?
m = 8 g = 0.008 kg
v = 400 m/s
1
Formula: 𝐸𝑐 = 2 𝑚𝑣 2
1
Formula: 𝐸𝑐 = 2 0.008 𝑘𝑔(400 𝑚/𝑠)2
𝑚2
𝐸𝑐 = 640 𝑘𝑔. 2 = 640 𝐽
𝑠

Montaña Rusa
El dibujo de una montaña rusa en un plano se puede interpretar como la representación de la
función energía potencial 𝑬𝒑 de un cuerpo en el campo gravitatorio. Cuanto más sube un móvil
la montaña rusa, mayor es su energía potencial y menor su energía cinética 𝑬𝒄 , y por tanto se
desplaza más lento. En los máximos relativos de dicha función (los picos de la montaña rusa)
su energía potencial será más elevada que en los puntos de su entorno. Estos puntos se llamarán
puntos de equilibrio mecánico inestable, ya que si se deposita en ellos un objeto con 𝑣 = 0 por
poco que se desplace de ese punto, el objeto siempre tenderá a alejarse. Por otro lado, si lo
situamos en los mínimos de la función (los valles de la montaña rusa), el móvil que los
abandonase en uno u otro sentido siempre tenderá a volver hacia ellos, son los puntos llamados
puntos de equilibrio estable. Como la energía mecánica 𝑬 del cuerpo se conserva 𝑬 = 𝑬𝒄 +
𝑬𝑷 = 𝑬𝑪 𝐦𝐚𝐱 = 𝑬𝑷 𝒎𝒂𝒙, en la figura.

Animación de una bola en una montaña rusa

Péndulo
En el caso de un péndulo, cuyo movimiento puede alcanzar una altura 𝒉 medida a partir de su posición
más baja, también se puede comprobar la ley de conservación de la energía. En los puntos más altos
(altura h), donde la energía potencial es máxima, la velocidad del péndulo es nula y el movimiento
cambia de sentido. Por otro lado, la posición más baja, que se pude llamar 𝑷, será aquella con una mayor
energía cinética y velocidad máxima pero con una energía potencial mínima. La posición 𝑷 se podrá
tomar como origen de la energía potencial (se le puede asociar una energía potencial nula).

22
Animación de un péndulo que alcanza una altura h

Problema No 1.14: ¿A qué altura debe de estar elevada una maceta que tiene una masa de 5
Kg para que su energía potencial sea de 80 Julios?
Solución:
Nos piden la altura, luego lo primero será despejar la altura (h) de la fórmula:
𝐸𝑝 80 𝐽
ℎ= = = 1.63 𝑚
𝑚∗𝑔 5 𝑘𝑔 ∗ 9.81 𝑚/𝑠 2

Problema No 1.15: Si la energía potencial de una pelota de golf al ser golpeada es de 80 J,


¿cuál será su masa si alcanza una altura de 30m?
Solución:
Tendremos que despejar de la fórmula la masa:
𝐸𝑝 80 𝐽
𝑚= = = 0.271.8 𝑘𝑔 = 271.8 𝑔
ℎ∗𝑔 30 ∗ 9.81 𝑚/𝑠 2

Problema No 1.16: Subimos un objeto de 12 Kg y lo subimos por una rampa inclinada 30º
una distancia de 14 metros. ¿Qué energía potencial tendrá al llegar arriba?

Solución:
Cuidado en este caso los 14 metros son la longitud de la parte inclinada, NO la altura vertical a
la que estará al llegar arriba. Lo que tendremos es que calcular la altura mediante trigonometría.

En un triángulo rectángulo el cateto h es igual a la hipotenusa (rampa) por el seno del ángulo
que forman. En nuestro caso el seno de 30º que es 1/2 = 0,5.
Luego
ℎ = 𝑑 ∗ 𝑆𝑒𝑛 30° = 14 ∗ 0.5 = 7 𝑚. Esta será la altura real a la que subimos el objeto.
Ahora calculamos la Ep.
𝑚
𝐸𝑝 = 𝑚𝑔ℎ = 12 𝑘𝑔 ∗ 9.81 2 ∗ 7 𝑚 = 824.04 𝐽
𝑠

23
Problema No 1.17: Un cuerpo de masa 40 kg resbala por el plano inclinado y liso y llega al
suelo con una velocidad de 20 m/s. Si mediante la ley de conservación de energía determinamos
que la energía potencial inicial es la misma que la energía cinética final. Determinar:
a. La energía potencial inicial
b. La altura en la que se encontraba el cuerpo
Datos: m = 40 kg, Vo = 0 m/s; g = 9.8 m/s2; Vf = 20 m/s
1 1 𝑚 2
a. 𝐸𝑐𝑓 = 2 𝑚 ∗ 𝑉𝑓2 = 2 (40 𝑘𝑔) (20 𝑠 ) = 8000 𝐽
𝐸𝑃𝑜 = 𝐸𝐶𝑓
𝐸𝑃𝑜 = 8000 𝐽
𝐸𝑃𝑜 = 𝑚 ∗ 𝑔 ∗ ℎ despejando h

b.
𝐸𝑝0 8000 𝑘𝑔 𝑚2 /𝑠 2
ℎ= = = 20.41 𝑚
𝑚 ∗ 𝑔 (40 𝑘𝑔)(9.8 𝑚/𝑠 2 )

Problema No 1.18: ¿Qué energía potencial tiene un ascensor de 800 Kg en la parte superior de
un edificio, a 380 m sobre el suelo? Suponga que la energía potencial en el suelo es 0.
Se tiene el valor de la altura y la masa del ascensor. De la definición de la energía potencial
gravitatoria:
𝑚
𝐸𝑝 = 𝑚𝑔ℎ = 800 𝑘𝑔 (9.8 ) (380 𝑚) = 2979,2 𝐽 = 2.9792 𝑀𝐽
𝑠2
Problema No 1.19: Un horno de microondas de 12 Kg se empuja para subirlo 14 m de una
superficie de una rampa inclinada 37º sobre la horizontal aplicando una fuerza constante de 120
N y paralela a la rampa.
El coeficiente de fricción cinética entre el horno y la rampa es de 0.25. a) ¿Qué trabajo realiza
la fuerza sobre el horno?; b) ¿Y la fuerza de fricción?; c) Calcule el aumento de energía
potencial del horno.

a) El trabajo de la fuerza está dado por el producto de la magnitud de la fuerza por la distancia
desplazada:
W = (120 N) (14 m) = 1680 J

24
b) El valor de la fuerza de fricción no es un dato dado del problema. Para determinarlo, se debe
hacer un DCL:

A partir de éste nuevo marco de referencia:


∑ Fy = 0 (Debido a que no hay desplazamiento en éste eje).
N - w cos 37º = 0
N = w cos 37º = m * g * cos 37º.
La fuerza de fricción es µk*N, entonces:
Wf = Ff * d = µk * N * d
Sustituyendo:
Wf = µk * m * g * cos 37º * d
Wf = (0.25) (12 Kg) (9.8 m/s^2) (14 m) (cos 37º) = 328.72 J
c) El aumento de energía potencial está dado por:
∆Ug = U(2) - U(1) = m*g*h(2) - m*g*h(1)
Si h(1) = 0 y h(2) = h:
∆Ug = m*g*h
Por trigonometría, sabemos que h = d * sen 37º:
∆Ug = m*g*d*sen 37º
∆Ug = (12 Kg) (9.8 m/s^2) (14 m) (sen 37º) = 990.83 J
Note que los maros de referencia para b) y c) son distintos: cuando se trabaja con energía
potencial, sólo interesa datos de altura.

Ejemplo 1.20: Un elevador con una masa de 2 500 kg descansa en un nivel a 10 m sobre la
base del pozo de un elevador. Cuando se eleva a 100 m sobre la base del pozo, se rompe el
cable que lo sostiene. El elevador cae libremente hacia la base del pozo y golpea contra un
fuerte resorte. El resorte está diseñado para poner al elevador en reposo y, por medio de un
diseño de retención, mantener al elevador en la posición de máxima compresión del resorte. Si
se supone que todo el proceso es sin fricción, y tomando g = 9.8 m/s2, calcule:
a. La energía potencial del elevador en su posición inicial con respecto a la base del pozo.
b. El trabajo hecho al subir el elevador.
c. e) La energía potencial del elevador en su posición más alta con respecto a la base del
pozo.
d. La velocidad y la energía cinética del elevador justo antes de que golpee en el resorte.
e. La energía potencial del resorte comprimido.
f. La energía del sistema que se compone del elevador y el resorte: 1) al inicio del proceso,
2) cuando el elevador alcanza su altura máxima, 3) justo antes de que el elevador incida
en el resorte, 4) después de que el elevador ha llegado al reposo.

Solución

25
Suponga que el subíndice 1 denota las condiciones iniciales; el subíndice 2, las condiciones
cuando el elevador está en su posición más alta, y el subíndice 3, las condiciones justo antes de
que el elevador llegue al resorte.
a) Por la ecuación (1.7), Ep1 = mz1g = (2500) (10) (9.8) = 245 000 J

b) Por la ecuación (1.1), 𝑊 = 𝑚𝑔(𝑧2 − 𝑧1 ) de donde


W = (2500)(9.8)(100 - 10) = 2 205 000 J

c) Por la ecuación (1.7), 𝐸𝑝2 = 𝑚𝑧2 𝑔 = (2500)(100)(9.8) = 2 450 000 J


Observe que 𝑊 = 𝐸𝑝2 − 𝐸𝑝1

d) Del principio de conservación de energía mecánica se puede escribir que la suma de los
cambios de las energías cinética y potencial durante los procesos desde las condiciones 2 a 3 es
cero; es decir
∆𝐸𝐶2→ 3 + ∆𝐸𝑃2→ 3 = 0 o 𝐸𝐶3 − 𝐸𝐶2 + 𝐸𝑃3 − 𝐸𝑃2 = 0
Sin embargo, 𝐸𝐶2 y 𝐸𝑃3 son cero. Por consiguiente,
𝐸𝐶3 = 𝐸𝑃2 = 2 450 000 𝐽
𝑚𝑢32 2𝐸𝐶3 (2)(2 450 000)
Con 𝐸𝐶3 = 𝑢32 = =
2 𝑚 2500

De donde, 𝑢3 = 44.27 m/s

e) Puesto que los cambios en la energía potencial del resorte y en la energía cinética del elevador
deben ser cero,

∆𝐸𝑃 (𝑟𝑒𝑠𝑜𝑟𝑡𝑒) + ∆𝐸𝐶 (𝑒𝑙𝑒𝑣𝑎𝑑𝑜𝑟) = 0

La energía potencial inicial del resorte y la energía cinética final del elevador son cero; por lo
tanto, la energía potencial final del resorte debe ser igual a la energía cinética del elevador justo
antes de que incida con el resorte. Así, la energía potencial final del resorte es 2 450 000 J.

f) Si el elevador y el resorte juntos se consideran como el sistema, la energía inicial del sistema
es la energía potencial del elevador, o 245 000 J. La energía total del sistema sólo cambia si se
transfiere trabajo entre él y los alrededores. Conforme se levanta el elevador se hace trabajo
sobre el sistema por acción de los alrededores en una cantidad de 2 205 000 J. De esta forma,
la energía del sistema cuando el elevador alcanza su altura máxima es 245 000 + 2 205 000 = 2
450 000 J. Los cambios siguientes ocurren por completo dentro del sistema, sin transferencia
de trabajo entre el sistema y los alrededores. Por lo tanto, la energía total del sistema permanece
constante en 2450000 J. Ésta sólo cambia de energía potencial de posición (elevación) del
elevador a energía cinética del elevador, y a energía potencial de la configuración en el resorte.
Este ejemplo ilustra el uso de la ley de conservación de la energía mecánica. Sin embargo, se
supone que todo el proceso ocurre sin fricción; los resultados obtenidos son exactos sólo para
procesos tan idealizados como éste

Energía mecánica: La energía mecánica se puede definir como la capacidad de producir un


trabajo mecánico el cual posee un cuerpo debido a causas de origen mecánico, como su posición
o su velocidad. Existen dos formas de energía mecánica que son la energía cinética y la energía
potencial.

26
La energía mecánica se puede definir como la capacidad de producir un trabajo mecánico el
cual posee un cuerpo debido a causas de origen mecánico, como su posición o su velocidad.
Existen dos formas de energía mecánica que son la energía cinética y la energía potencial.

Conservación de la energía mecánica


Para sistemas abiertos formados por partículas que interactúan mediante fuerzas puramente
mecánicas o campos conservativos la energía se mantiene constante con el tiempo:

𝐸𝑚 = 𝐸𝑝 + 𝐸𝑐

Es importante notar que la energía mecánica así definida permanece constante si únicamente
actúan fuerzas conservativas sobre las partículas. Sin embargo, existen ejemplos de sistemas de
partículas donde la energía mecánica no se conserva.

ENERGÍA ELÉCTRICA
Ley del Ohm

La energía eléctrica es la forma de energía que resultará de la existencia de una diferencia de potencial
entre dos puntos, situación que permitirá establecer una corriente eléctrica entre ambos puntos si se los
coloca en contacto por intermedio de un conductor eléctrico para obtener el trabajo mencionado.

Se denomina energía eléctrica a la forma de energía que resulta de la existencia de una diferencia de
potencial entre dos puntos, lo que permite establecer una corriente eléctrica entre ambos cuando se los
pone en contacto por medio de un conductor eléctrico. La energía eléctrica puede transformarse en
muchas otras formas de energía, tales como la energía lumínica o luz, la energía mecánica y la energía
térmica.
a) Si la magnitud a calcular es la Intensidad (I):

b) Si la magnitud a calcular es la tensión, voltaje (V) o diferencia de potencial (d.d.p.):

27
c) Si la magnitud a calcular es la resistencia (R):

Resistencia Eléctrica:
- “Es la mayor o menor dificultad que presenta un material al paso de la corriente
eléctrica”.
- Su unidad de medida es el Ohmios, (Ω).
- Se mide con el ohmímetro.

La resistencia (R) de un conductor depende de las características del material, es decir, de


su resistividad (ρ), así como de la longitud (l) y la sección del conductor (S).
Todos estos parámetros se relacionan mediante la expresión:

R = resistencia, (Ω),
ρ= resistividad, (Ω·m).
l = longitud, (m).
S= sección, (m²).

Recuerda

28
La estructura atómica de cada material determina la mayor o menor facilidad con que se
desplazan los electrones. Los materiales se pueden clasificar en:
 Conductores: se trata de materiales que permiten la circulación de electrones. Son
buenos conductores el cobre, la oro, la plata, el aluminio y en general todos los metales.
 Aislantes: en estos materiales, los átomos retienen los electrones, por lo que no pueden
circular libremente, luego son malos conductores de la electricidad. Ejemplos de
materiales aislantes son: la madera, el vidrio, el plástico, el aire …
 Semiconductores: presentan propiedades intermedias entre los materiales conductores
y los aislantes. Los más importantes son el germanio y el silicio (componentes
electrónicos).

Ley De Ohm
El físico alemán Georg Simón Ohm descubrió a principios del siglo XIX, que en un circuito
la Intensidad, la Resistencia y la Tensión se relacionan según la ley que lleva su nombre, cuya
expresión es:

Ejemplo 1.21

Ejemplo 1.22

29
Ejemplo 1.23

Es aquella energía que se debe al movimiento de los electrones, comprende tanto voltaje como
amperaje. Puesto que una partícula cargada eléctricamente puede generar una fuerza sobre
ciertos objetos de su vecindad, se le denomina campo eléctrico, se expresa por la siguiente
ecuación

𝑃(𝐸𝑒 ) = 𝑉 ∗ 𝐼
𝑃
𝑉=
𝐼
𝑃
𝐼=
𝑉
Donde
𝑃 = 𝑃𝑜𝑡𝑒𝑛𝑐𝑖𝑎
𝑉 = 𝑉𝑜𝑙𝑡𝑎𝑗𝑒
𝐼 = 𝐶𝑎𝑟𝑔𝑎 𝑒𝑙𝑒𝑐𝑡𝑟𝑖𝑐𝑎 𝑜 𝐼𝑛𝑡𝑒𝑛𝑠𝑖𝑑𝑎𝑑

30
Intensidad de Corriente Eléctrica (I):
 Se define como “la cantidad de carga eléctrica (número de electrones) que circula por el
circuito en la unidad de tiempo (el segundo)”.
 Su unidad de medida es el Amperio, (A).
 Se mide con el amperímetro.

Tensión, Voltaje (V) O Diferencia de Potencial (ddp):


 “Indica la diferencia de energía entre dos puntos de un circuito”. Esta magnitud es una
medida de la fuerza que hay que comunicar a los electrones para que se muevan a lo largo
del circuito.
 Su unidad de medida el voltio (v).
 Se mide con el voltímetro.

Potencia ( P )
Todos los receptores eléctricos trasforman la energía eléctrica en otras formas de energía (calor,
movimiento, sonido).
 "La potencia es la magnitud que mide la energía consumida por un aparato eléctrico en una
unidad de tiempo".
 Su unidad de medida es el vatio, (W)
 Se mide con el vatímetro.

Ejemplo 1.24

31
Ejemplo 1.25
En el cargador de un móvil viene marcado 230 V - 25 mA.
a) Calcula su potencia en vatios.
b) Si lo pones a cargar todas las noches durante 8 horas, ¿Cuánta energía consume, en kWh,
al año?
c) Si el coste es de 0,125 €/kWh, ¿cuánto cuesta cargarlo durante todo un año?

Si tu equipo de música es de 40 W y estás escuchando música durante 5 horas, ¿cuántos kWh


has consumido? Sabiendo que 1 kWh cuesta actualmente 0,125 € ¿cuánto nos costará escuchar
música?

LA ENERGÍA ELÉCTRICA (E)


- " Es la Energía consumida por un receptor en un circuito eléctrico".
- Se mide en Julios (J) y está relacionada con la Potencia mediante la siguiente expresión:

Donde:
E = energía eléctrica (Julios, J) à 1 Julio (J) = vatio x segundo à 1 J = W x s
P = potencia (vatios, W)
t = tiempo (segundos, s)
Ejemplo No

Ejercicios Propuestos
Ejercicio 1: ¿Cuál es la resistencia de un circuito por el que circula una corriente eléctrica
de 2 amperios con una tensión de 12 voltios?

Ejercicio 2: Calcula la intensidad de la corriente que circula por un dispositivo de 2.000Ω de


resistencia al aplicarle una diferencia de potencial de 200 V.

Ejercicio 3: Calcula el voltaje existente entre los extremos de una resistencia de 100 Ω por la
que circula 0,1 A.

32
Ejercicio 4: Si por una resistencia de 15 Ω circula una intensidad de 30 A, ¿qué diferencia de
potencial se creará?

Ejercicio 5: Si en un conductor tiene en sus extremos una diferencia de potencial de 220 V y


su resistencia es de 100 Ω. ¿Qué intensidad circula a través de él y en qué sentido?

Ejercicio 6: ¿Cuál es la resistencia de una bombilla de 100 W y 230 V? ¿Qué intensidad circula
por ella si la conectamos a 125 V?

Ejercicio 7: Calcula el valor de la resistencia R en este circuito. ¿Qué intensidad circularía si


se duplicara el valor de R ?

Ejercicio 8: Calcula la intensidad de la corriente en este circuito.


a) Qué marcaría el amperímetro si el valor de la resistencia se redujera a la mitad.

Ejercicio 9: ¿Qué valor marcará el voltímetro en bornes de la resistencia de 100 Ω?.

Ejercicio 10: Si tu equipo de música es de 40 W y estás escuchando música durante 5 horas,


¿cuántos kWh has consumido? Sabiendo que 1 kWh cuesta actualmente 0,125 € ¿cuánto nos
costará escuchar música?

Ejercicio 11: En el cargador de un móvil viene marcado 230 V - 25 mA.


a) Calcula su potencia en vatios.
b) Si lo pones a cargar todas las noches durante 8 horas, ¿Cuánta energía consume, en kWh, al
año?
c) Si el coste es de 0,125 €/kWh, ¿cuánto cuesta cargarlo durante todo un año?

Calor

33
Sabemos por experiencia que un objeto caliente que se pone en contacto con otro objeto frío
tiende a enfriarse, al mismo tiempo que este último se calienta. Una visión razonable es que
algo se transfiere del objeto caliente al frío, y a ese algo le llamamos calor Q. En estos términos
se dice que el calor siempre fluye de una temperatura más alta a una más baja, lo cual nos lleva
al concepto de temperatura como la fuerza impulsora para la transferencia de energía como
calor. Con más precisión, la rapidez de transferencia de calor de un cuerpo a otro es
proporcional a la diferencia de temperatura entre los dos cuerpos; cuando no existe diferencia
de temperatura, no hay transferencia neta de calor. En sentido termodinámico, el calor nunca
se considera como algo que está almacenado dentro de un cuerpo. Al igual que el trabajo,
solamente existe como energía en tránsito desde un cuerpo hacia otro; o, en términos
termodinámicos, entre un sistema y sus alrededores. Cuando se agrega energía en forma de
calor a un cuerpo, se almacena no como calor sino como energías cinética y potencial de los
átomos y de las moléculas que constituyen el sistema.

A pesar de la naturaleza transitoria del calor, con frecuencia se le ve sólo desde el punto de vista
de su efecto sobre el cuerpo del cual o al cual se transfiere. De hecho, alrededor de 1930 las
definiciones de unidades de calor estaban basadas en los cambios de temperatura de una masa
de una unidad de agua. De esta forma, la caloría se definió por mucho tiempo como la cantidad
de calor que, al transferirse a un gramo de agua, elevaba su temperatura un grado Celsius. Del
mismo modo, la unidad térmica inglesa (o Btu, British termal unit) se definió como la cantidad
de calor que, al transferirse a una libra masa de agua, elevaba su temperatura en un grado
Fahrenheit. Aunque estas definiciones proporcionan una "sensación" de la magnitud de estas
unidades de calor, dependen de experimentos realizados con agua y, por lo tanto, están sujetas
a cambios en la medida en que las mediciones se vuelven más precisas. En la actualidad se
reconoce a la caloría y al (Btu) como unidades de energía, y se definen con respecto al joule, la
unidad de energía del SI, que es igual a 1 N m. Éste es el trabajo mecánico realizado por la
fuerza de un newton que actúa a través de una distancia de un metro. Todas las demás unidades
de energía se definen como múltiplos del joule. El pie-libra fuerza, por ejemplo, es equivalente
a 1.3558179 J, la caloría es igual a 4.1840 J, mientras que el (Btu) es igual a 1055.04 J. La
unidad SI para la potencia es el watt, cuyo símbolo es W y se define como una relación de
energía de un joule por segundo. La tabla A.1 en el apéndice A proporciona una lista amplia de
factores de conversión para la energía, así como para otras unidades

Energía Calorífica
La energía térmica (también energía calórica o energía calorífica) es la manifestación de
la energía en forma de calor. En todos los materiales los átomos que forman sus moléculas
están en continuo movimiento ya sea trasladándose o vibrando.

La energía térmica es la parte de la energía interna de un sistema termodinámico en equilibrio


que es proporcional a su temperatura absoluta y se incrementa o disminuye por transferencia de
energía, generalmente en forma de calor o trabajo, en procesos termodinámicos. A nivel
microscópico y en el marco de la Teoría cinética, es el total de la energía cinética media presente
como el resultado de los movimientos aleatorios de átomos y moléculas o agitación térmica,
que desaparecen en el cero absoluto.

La energía térmica (también energía calórica o energía calorífica) es la manifestación de la


energía en forma de calor. En todos los materiales los átomos que forman sus moléculas están
en continuo movimiento ya sea trasladándose o vibrando. Este movimiento implica que los
átomos tengan una determinada energía cinética a la que nosotros llamamos calor, energía
térmica o energía calorífica.

34
Si se aumenta temperatura a un elemento aumenta su energía térmica; pero no siempre que se
aumenta la energía térmica de un cuerpo aumenta su temperatura ya que en los cambios de fase
(de líquido a gas, por ejemplo) la temperatura se mantiene. Por ejemplo, al calentar un cazo de
agua, poco a poco le vamos dando energía térmica y va aumentando su temperatura, pero
cuando llega a los 100ºC (temperatura de ebullición) la energía térmica que le suministramos a
partir de este momento se utiliza para cambiar de fase (de líquido a gas, es decir, a vapor de
agua) pero no para aumentar su temperatura.

Transmisión de la energía térmica


La energía térmica puede transmitirse de un cuerpo a otro siguiendo las leyes de la
termodinámica de tres modos distintos:
 Transmisión de calor por radiación.
 Transmisión de energía térmica por conducción.
 Transmisión de energía calorífica por convección.

La energía térmica por radiación se transmite a través de ondas electromagnéticas. Es el


modo con el que nos llega la energía térmica proveniente del Sol. El principal ejemplo de este
caso lo encontramos en las instalaciones de energía solar térmica, que aprovechan la radiación
solar para calentar agua. El agua caliente que se obtiene de estas instalaciones solares
habitualmente se utiliza para calefacción y para usarla como agua caliente sanitaria.

La transmisión de la energía térmica por conducción se experimenta cuando un cuerpo


caliente está en contacto físico con otro cuerpo más frío. La energía se transmite siempre del
cuerpo caliente al cuerpo frío. Si ambos cuerpos están a la misma temperatura no hay
transferencia energética. Cuando tocamos un trozo de hielo con la mano parte de la energía
térmica de nuestra mano se transfiere al hielo, por eso tenemos sensación de frío.

La transmisión de la energía térmica por convección se produce cuando se trasladan las


moléculas calientes de un lado a otro. Sería el caso del viento, capaz de mover moléculas con
cierta energía calorífica de un lado a otro.

Medida de la capacidad calorífica


Para medir la capacidad calorífica bajo unas determinadas condiciones es necesario comparar
el calor absorbido por una sustancia (o un sistema) con el incremento de temperatura resultante.
La capacidad calorífica viene dada por:
Q
C = lim∆t→0
∆T

 C es la capacidad calorífica, que en general será función de las variables de estado.


 Q es el calor absorbido por el sistema.
 ∆T la variación de temperatura
Se mide en unidades del SI julios por kelvin (J/K) (o también en cal/°C).
La capacidad calorífica (C) de un sistema físico depende de la cantidad de sustancia o masa de
dicho sistema. Para un sistema formado por una sola sustancia homogénea se define además
el calor específico o capacidad calorífica específica (c) a partir de la relación:

Q
C= = c∗m
∆T
Donde
 C es la capacidad calorífica del cuerpo o sistema
 c es el calor específico o capacidad calorífica específica

35
 m la masa de sustancia considerada
De las anteriores relaciones es fácil inferir que al aumentar la masa de una sustancia, se aumenta
su capacidad calorífica ya que aumenta la inercia térmica, y con ello aumenta la dificultad de
la sustancia para variar su temperatura. Un ejemplo de esto se puede apreciar en las ciudades
costeras donde el mar actúa como un gran termostato regulando las variaciones de temperatura.

En general, conociendo la masa de sustancia, m, la variación de temperatura que experimenta,


ΔT, y su capacidad calorífica específica se puede calcular la energía calorífica intercambiada o
transferencia de calor, Q, según la fórmula:

Q=m·c·ΔT
Donde:
m = masa de la sustancia
c = capacidad calorífica específica (J/K·kg)
ΔT = Tf – Ti (temperatura final menos temperatura inicial del sistema)

Por ejemplo, para calentar una barra de 6kg de aluminio de 25ºC a 40ºC, sabiendo que la C
del aluminio es de 895 J/kg·K
Q = m·c·ΔT = 6 kg · 895 (J/K·kg) · (40-25) = 80550 J

Capacidad calorífica molar de una sustancia


La Capacidad calorífica molar, Cm, es la energía calorífica necesaria para aumentar 1K o 1ºC
la temperatura de un mol de cualquier sustancia. En este caso, la transferencia de calor será:
Q= n·ΔT·Cm
Donde:
n = número de moles
Cm = capacidad calorífica molar
ΔT = Tf – Ti (temperatura final menos temperatura inicial del sistema)
Cabe destacar que para el caso de sólidos y líquidos, la capacidad calorífica específica y la
capacidad calorífica molar son iguales tanto si el proceso se realiza a presión constante como
se se realiza a volumen constante. No obstante, en el caso de los gases, estos parámetros varían,
lo cual se indica con un subíndice v y p para el proceso a V=cte y P=cte respectivamente: Cv,
Cp, Cmv, Cmp, por lo que si en un ejercicio debemos calcular el calor transferido en el caso de
un gas nos deben indicar si el proceso es isocoro o isobárico.

Calor específico y capacidad calorífica de algunos materiales

36
Densidad
En física y química, la densidad (del latín densĭtas, -ātis) es una magnitud escalar referida a la cantidad
de masa en un determinado volumen de una sustancia. Usualmente se simboliza mediante la
letra rho ρ del alfabeto griego. La densidad media es la relación entre la masa de un cuerpo y el volumen
que ocupa.

𝒎
𝝆=
𝒗
Donde
𝐾𝑔
𝜌 = 𝐷𝑒𝑛𝑠𝑖𝑑𝑎𝑑, 𝑑𝑎𝑑𝑜 𝑒𝑛
𝑚3
𝑚 = 𝑀𝑎𝑠𝑎 𝑑𝑒𝑙 𝑐𝑢𝑒𝑟𝑝𝑜, 𝑑𝑎𝑑𝑜 𝑒𝑛 𝐾𝑔
𝑣 = 𝑉𝑜𝑙𝑢𝑚𝑒𝑛 𝑑𝑒𝑙 𝑐𝑢𝑒𝑟𝑝𝑜, 𝑑𝑎𝑑𝑜 𝑒𝑛 𝑚3

Peso especifico
Se llama peso específico al peso que posee una sustancia en un determinado volumen. El
símbolo del peso específico es 𝑃𝑒 . Suele expresarse en newtons sobre metro cúbico (N/m3).
El peso específico de un cuerpo o sustancia, es la relación que existe entre el peso y el volumen
que ocupa una sustancia ya sea en estado sólido, líquido o gaseoso. Es una constante en el
sentido de que es un valor que no cambia para cada sustancia ya que a medida que aumenta su
peso también aumentara su volumen ocupado, al igual que sucede con la densidad.
𝑷 𝒎𝒈
𝑷𝒆 = = = 𝝆𝒈
𝒗 𝒗
Donde
𝑃𝑒𝑠𝑜 𝑁
𝑃𝑒 = 𝑃𝑒𝑠𝑜 𝑒𝑠𝑝𝑒𝑐𝑖𝑓𝑖𝑐𝑜 = 𝑣𝑜𝑙𝑢𝑚𝑒𝑛=𝑚3
𝑃 = 𝑃𝑒𝑠𝑜 𝑑𝑒 𝑙𝑎 𝑠𝑢𝑠𝑡𝑎𝑛𝑐𝑖𝑎, 𝑑𝑎𝑑𝑜 𝑒𝑛 𝑁𝑒𝑤𝑡𝑜𝑛
𝑚 = 𝑀𝑎𝑠𝑎 𝑑𝑒𝑙 𝑐𝑢𝑒𝑟𝑝𝑜, 𝑑𝑎𝑑𝑜 𝑒𝑛 𝐾𝑔
𝑣 = 𝑉𝑜𝑙𝑢𝑚𝑒𝑛 𝑑𝑒𝑙 𝑐𝑢𝑒𝑟𝑝𝑜, 𝑑𝑎𝑑𝑜 𝑒𝑛 𝑚3
𝐾𝑔
𝜌 = 𝐷𝑒𝑛𝑠𝑖𝑑𝑎𝑑, 𝑑𝑎𝑑𝑜 𝑒𝑛 3
𝑚
𝑔 = 𝐴𝑐𝑒𝑙𝑒𝑟𝑎𝑐𝑖𝑜𝑛 𝑑𝑒 𝑙𝑎 𝑔𝑟𝑎𝑣𝑒𝑑𝑎𝑑

Esta constante tiene la importancia de ser una propiedad intensiva, ya que nos permitirá
identificar a la sustancia.

37
Conversión entre las unidades del USCS y el SI
Para convertir del sistema USCS al SI: Para convertir el valor de una variable que esté en
unidades de USCS a su equivalente en unidades del SI, multiplique el valor que desea convertir
por el valor que aparece al lado derecho de la equivalencia correspondiente en la Tabla de
equivalencias.
Ejemplo: Convierta una longitud L = 3.25 a su valor equivalente en milímetros.
Solución: La equivalencia que le corresponde es: 1.0 in = 25.4 mm
L = 3.25 in × (25.4 mm/in) = 82.55 mm
Para convertir unidades del SI al sistema USCS: Para convertir el valor de una variable de
unidades del SI a su equivalente en unidades del USCS, divida el valor que desea convertir
entre el valor asentado en el lado derecho de la equivalencia que le corresponde en la Tabla de
equivalencias.
Ejemplo: Convierta un área A = 1 000 mm2 a su equivalente en pulgadas cuadradas.
Solución: La equivalencia que le corresponde es: 1.0 in2 = 645.16 mm2
A = 1 000 mm2/(645.16 mm2/in2) = 1.55 in2

Sistema Internacional de unidades


El año de 1960 marcó el nacimiento del Sistema Internacional de unidades tal como se conoce
en la actualidad; este suceso ocurrió en la XI Conferencia General de Pesas y Medidas (CGPM)

38
bajo cuya autoridad funciona la Oficina Internacional de Pesas y Medidas (BIPM: Bureau
International des oids et Mesures), la cual se llevó a cabo en Sévres, Francia.
El Sistema Internacional de unidades, universalmente abreviado SI, del francés Le Système
International d´Unitès, es el moderno sistema métrico de medición más utilizado en la
ciencia y en el comercio internacional.

El sistema métrico anterior, MKSA (metro, kilogramo, segundo y ampere), desarrollado por
Giovani Giorgi, había logrado ya coherencia entre el metro, el kilogramo-masa, el segundo y
las unidades prácticas de electricidad: volt, ampere, joule, watt, ohm, coulomb y henry.

Debido al grado de perfección que había alcanzado Giorgi con dicho sistema, fue elegido para
cimentar el Sistema Internacional.
La versión original del Sistema Internacional está cimentada en siete unidades fundamentales
y dos suplementarias; además, define 22 unidades derivadas, aunque son muchas más las que
se establecen simplemente como consecuencia y por la sola aplicación de las leyes de la física
y de los principios del antiguo sistema métrico.

En la definición oficial del SI se citan algunas unidades ajenas al sistema, las cuales no son
homogéneas, por lo menos en esta etapa de desarrollo del sistema.
En algunos casos, los científicos que afinaron la presentación original del SI se vieron forzados
a respetar la costumbre, como ocurre con las unidades antiguas para los ángulos, el quilate, la
hora, el minuto y el watt-hora.

El SI divide en tres clases a las unidades: unidades de base; unidades derivadas y unidades
suplementarias, las cuales, en conjunto, forman el sistema de unidades coherentes del SI,
además de que se incluyen prefijos para formar múltiplos y submúltiplos decimales.
Entre los organismos de rango internacional que más se han destacado en el estudio de la
normalización de unidades y las magnitudes físicas se encuentra la Conferencia General de
Pesas y
Medidas (CGPM) y el Comité Internacional de Pesas y Medidas (CIPM).
Asimismo, se han creado organismos no gubernamentales cuyo propósito es la unificación de
conceptos y unidades y el desarrollo de disciplinas especializadas.

Unidades básicas y derivadas


El SI tiene una lista de unidades de base definidas en forma absoluta, sin que estén referidas a
cualquier otra unidad; las unidades de base son consistentes con la parte del sistema métrico
llamado sistema MKS.
Como ya se mencionó, el SI cuenta con siete unidades de base: el metro para longitud, el
kilogramo para masa, el segundo para tiempo, el ampere para corriente eléctrica, el kelvin
para temperatura, el mol para cantidad de sustancia y la candela para intensidad de luz (véase
tabla 2.4).

39
Originalmente, las medidas de base, o fundamentales, recibían este nombre porque se
consideraba que éstas eran independientes entre sí y permitían, a su vez, la definición de otras
unidades. Los patrones correspondientes eran medidas materializadas que se conservaban en
lugares acordados y bajo condiciones determinadas. Los avances científicos y técnicos, así
como la disponibilidad de instrumentos de mayor exactitud han dado por resultado que, con
excepción del kilogramo, las unidades de base se definan actualmente de diferente forma: con
base en experimentos físicos.

 Unidad SI de masa: kilogramo (kg) Masa del prototipo internacional de kilogramo que se
conserva en la Oficina Internacional de Pesas y Medidas de Sévres (París)
 Unidad SI de tiempo: segundo (s) Es la duración de 9.192.631.770 periodos de la radiación
correspondiente a la transición entre los dos niveles hiperfinos del estado fundamental de
átomo de cesio 133
 Unidad SI de temperatura: kelvin (K) Es la fracción 1/273,16 de la temperatura
termodinámica del punto triple del agua
 Unidad SI de intensidad de corriente: amperio (A) Intensidad de una corriente constante
que, mantenida en dos conductores paralelos rectilíneos de longitud infinita y sección
circular despreciable, colocados a una distancia de 1 m el uno del otro, en el vacío, produce
entre estos dos conductores una fuerza de 2x10-7 newton por metro de longitud
• Unidad SI de cantidad de sustancia: mol (mol) Es la cantidad de sustancia que contiene
tantas entidades elementales como átomos hay en 0,012 kg de carbono 12. Las entidades
elementales deben ser especificadas: átomos, moléculas, iones, electrones u otras partículas
o agrupamientos especificados de tales partículas
• Unidad SI de intensidad luminosa: candela (cd) Es la intensidad luminosa, en una dirección
dada, de una fuente que emite energía radiante monocromática de 540x1012 Hz de
frecuencia, y que tiene una intensidad radiante en dicha dirección de 1/683 vatios de
estereorradián.

Las unidades derivadas se expresan algebraicamente en términos de las unidades de base o de


otras unidades derivadas (incluyendo el radián y el estereorradián). Los símbolos para unidades
derivadas son obtenidos a través de operaciones de multiplicación y división. Así, por ejemplo,
la unidad derivada de masa molar (masa de un kmol de un átomo o molécula) es el kilogramo
por kmol (kg/kmol); o la unidad de fuerza: el newton, es definido como la fuerza que permite
acelerar una masa de un kilogramo en un metro por segundo por segundo; esto significa que el
newton es igual a un kilogramo metro por segundo cuadrado, así la relación es N = kg m/s2

Las unidades derivadas del SI incluyen el radián y el estereorradián, para ángulos planos y
sólidos, respectivamente; newton para fuerza, pascal para presión, joule para energía, watt
para potencia, grado Celsius para mediciones cotidianas de temperatura.

40
Por su parte, las unidades para medición de electricidad son coulomb (carga), volt (potencial),
faraday (capacitancia), ohm (resistencia) y siemens (conductancia); las unidades para medir
el magnetismo son weber (flujo magnético), tesla (densidad de flujo magnético) y henry
(inductancia), lumen para flujo luminoso, lux para iluminación, Hertz para frecuencia,
becquerel para índices de radiactividad y otros eventos aleatorios, gray y sievert para dosis de
radiación y katal, que es una unidad de actividad catalítica usada en bioquímica. Estas unidades
derivadas del SI, además de nombres especiales, poseen símbolos exclusivos (véase tabla 2.5).
Por otra parte, en la tabla 2.6 se pueden observar las magnitudes derivadas más utilizadas, junto
con sus unidades del SI y sus símbolos.

Múltiplos y submúltiplos de unidades


Para escribir números grandes se multiplican múltiplos de diez, de modo que cien es diez por
diez, y se escribe 100; así, mil es diez por diez por diez, y se escribe 1 000; etcétera. El número

41
de ceros que aparece en cada uno de estos números es igual al número de veces que se
multiplican los dieces.

Todas las unidades del SI se derivan de unidades fundamentales; las cantidades mayores y más
pequeñas se expresan agregando prefijos adecuados a la unidad fundamental, por ejemplo las
distancias de las carreteras se expresan en kilómetro, que es exactamente 1000 m (1 ×103 m).
Los prefijos fueron tomados del griego y del latín de acuerdo con las tablas 2.7a y 2.7b.

Ahora bien, si se reservan los prefijos griegos para las unidades grandes y los latinos para las
pequeñas, para el caso de medidas de longitud se tiene:

Al hacer que todos los submúltiplos varíen según potencias de diez, se hace muy fácil convertir
una unidad en otra (debido a que nuestro sistema de numeración es decimal).
Además, una vez memorizado los prefijos, servirán para cualquier tipo de medición.
De esta forma, si se menciona que un poise es una unidad de viscosidad, no importa cuán grande
sea dicha unidad ni de qué manera esté relacionada con otro tipo de unidades y ni siquiera
interesa saber con exactitud qué es la viscosidad. A pesar de no saber absolutamente nada del
tema, uno sabe que un centipoise equivale a un céntimo de poise, que una hectárea es igual a
cien áreas, que un decibel es un décimo de bel; e incluso que un “kilopesos” equivale a mil
pesos.

En el sistema métrico decimal de 1795, los prefijos no sobrepasaron la marca de millar; como
consecuencia, en forma original el sistema métrico sólo muestra prefijos que cubren nada más
seis órdenes de magnitud. La unidad más grande, kilo, es un millón (10) de veces más grande
que la unidad más pequeña, mili, y el exponente 6 es el que indica los órdenes de magnitud.

Seis órdenes de magnitud pueden ser suficientes para la vida diaria, pero, a medida que los
instrumentos se desarrollaban, la ciencia requería expresar cantidades muy grandes y muy
pequeñas, en casi todos los campos de la medición, por lo que no quedaba más remedio que
extender el intervalo del sistema.
Así fue como se comenzaron a usar prefijos extraoficiales para las unidades que estaban por
encima de kilo y por debajo de mili y, por supuesto, con ello se corría el riesgo de crear
disidencias.

En 1958, el Comité Internacional de Pesas y Medidas de París acordó establecer un conjunto


más amplio de prefijos.

42
Para evitar una gran proliferación de estos prefijos, los factores varían en el orden de 103n (tres
órdenes de magnitud), excepto para los valores más próximos a la unidad, donde se manejan
otros prefijos que expresan relaciones de 10-2, 10-1, 10 y 102 de la unidad indicada.

Cabe resaltar que no deben utilizarse prefijos compuestos, ya que éstos, como es lógico, no son
absolutamente necesarios, aunque sí convenientes, porque evitan el manejo de valores
numéricos desmesuradamente grandes o pequeños y muchas veces facilitan la apreciación
cuantitativa de las magnitudes físicas. Cuando se manejan unidades del SI con prefijos en un
cálculo numérico, pueden necesitarse factores de conversión para obtener el resultado final en
la unidad que se desea, pero estos factores serán siempre potencias de diez.

A continuación se muestran los principales prefijos usados en el SI, junto con un par de los
prefijos más antiguos, los cuales se han incluido por razones de continuidad (véase tabla 2.8).

En 1962 se agregaron los prefijos fento, que representa un milmillonésimo (10-15) y ato que
equivale a un trillonésimo (10-18); ambos prefijos provienen de las palabras danesas femto
(“quince”) y atto (“dieciocho”). Así, se sabe que un picómetro es un billonésimo de metro, un
nanogramo es un milmillonésimo de gramo y un gigasegundo representa mil millones de
segundos (véase tabla 2.9).

43
Unidades que no son del SI
Las tablas 2.10 y 2.11 reúnen un grupo de unidades que pueden definirse con precisión en la
forma habitual del SI, pero que no pertenecen formalmente a este sistema. Las unidades que
aparecen en la primera de éstas (véase tabla 2.10) son múltiplos o fracciones decimales de
unidades del SI y las de la segunda (véase tabla 2.11) no guardan relación decimal respecto a
las unidades correspondientes del SI.
Con excepción de la atmósfera (que debe conservarse en física-química, porque a ésta se
refieren los estados estándar o de referencia), las unidades que no pertenecen al SI ya no son
necesarias y la mayor parte de las mismas deben considerarse en desuso.
Algunas unidades no pertenecientes al SI, como minuto, hora, grado, Celsius y, posiblemente,
litro, han de conservarse, porque como es natural, están muy difundidas por el uso cotidiano y
general. El grado Celsius continuará empleándose, sin duda, en algunos campos de las ciencias
físicas, como una unidad familiar para medir temperaturas termodinámicas, que excedan 273,15
K.
Algunas ocasiones pueden convenir expresar los datos experimentales en unidades que no son
del SI, en razón de los instrumentos de medida que todavía se utilizan.
Sin embargo, las unidades que no son del SI en general deben evitarse cuando se registren los
valores de las magnitudes físicas que se vayan calculando a partir de los datos experimentales.

44
Sistemas de Unidades
El sistema internacional oficial de unidades es el SI (Système International d’Unités). En la
actualidad se realizan grandes esfuerzos para su adopción universal como el sistema exclusivo,
tanto para las materias de ingeniería y las científicas; pero los sistemas más antiguos,
particularmente los sistemas centímetro-gramo-segundo (cgs) y de ingeniería gravitacional
pie-libra-segundo (fps), todavía se usan y probablemente continuarán utilizándose por algún
tiempo.
El ingeniero químico encuentra muchos datos fisicoquímicos expresados en unidades cgs;
aunque muchos de los cálculos se realizan de una forma más conveniente en unidades fps.
Por otra parte, las unidades del SI alcanzan un uso creciente tanto en ciencia como en ingeniería.
Así que es imperativo convertirse en un experto en el uso de los tres sistemas.
En el tratamiento que sigue, se estudia primero el sistema SI y posteriormente se derivan los
demás sistemas a partir de él. Sin embargo, el proceso histórico ha sido al contrario, ya que las
unidades SI evolucionaron a partir del sistema cgs. Debido a la importancia creciente del
sistema SI, debería tener lógicamente la preferencia. Si, con el tiempo, los otros sistemas

45
desaparecen progresivamente, habrán de ignorarse para utilizar de manera exclusiva el sistema
SI
.
Cantidades físicas
Toda cantidad física consta de dos partes: una unidad, que expresa la cantidad de que se trata y
da el estándar para su medida, y un número, que indica cuántas unidades se necesitan para
completar la cantidad. Por ejemplo, la afirmación de que la distancia entre dos puntos es 3 m
expresa lo siguiente: se ha medido una longitud determinada; para medirla se ha elegido una
unidad de longitud estándar, denominada metro; y para cubrir la distancia desde un extremo
hasta el otro se necesitan tres unidades de 1 m. Si un número entero de unidades resulta
demasiado pequeño o demasiado grande para cubrir una distancia determinada, se definen
submúltiplos, que son fracciones de la unidad, de manera que sea posible realizar la medida con
cualquier grado de precisión en términos de las unidades fraccionarias. Ninguna cantidad física
está definida mientras no se proporcionen tanto el número como la unidad.

Unidades SI
El sistema SI cubre todo el campo de la ciencia y la ingeniería, incluyendo el electromagnetismo
y la iluminación. Para los propósitos de este libro, es suficiente un subconjunto de unidades SI
que comprendan la química, la gravedad, la mecánica y la termodinámica. Las unidades son
derivables de: 1) cuatro proporcionalidades de química y física; 2) estándares arbitrarios para
la masa, la longitud, el tiempo, la temperatura y el mol, y 3) elecciones arbitrarias para los
valores numéricos de dos constantes de proporcionalidad.

46
47
Caballo de fuerza mecánico. El caballo de vapor eléctrico se toma para que sea exactamente igual a 746 W.

48
49
Peso
En física clásica, el peso es una medida de la fuerza gravitatoria que actúa sobre un objeto. El peso
equivale a la fuerza que ejerce un cuerpo sobre un punto de apoyo, originada por la acción del campo
gravitatorio local sobre la masa del cuerpo. Por ser una fuerza, el peso se representa como un vector,
definido por su módulo, dirección y sentido, aplicado en el centro de gravedad del cuerpo y dirigido
aproximadamente hacia el centro de la Tierra. Por extensión de esta definición, también podemos
referirnos al peso de un cuerpo en cualquier otro astro (Luna, Marte,...) en cuyas proximidades se
encuentre.

La magnitud del peso de un objeto, desde la definición operacional de peso, depende tan solo de la
intensidad del campo gravitatorio local y de la masa del cuerpo, en un sentido estricto. Sin embargo,
desde un punto de vista legal y práctico, se establece que el peso, cuando el sistema de referencia es la
Tierra, comprende no solo la fuerza gravitatoria local, sino también la fuerza centrífuga local debido a
la rotación de la Tierra; por el contrario, el empuje atmosférico no se incluye, ni ninguna otra fuerza
externa.

Peso y masa
Peso y masa son dos conceptos y magnitudes físicas muy diferentes, aunque aún en estos
momentos, en el habla cotidiana, el término “peso” se utiliza a menudo erróneamente como
sinónimo de masa, la cual es una magnitud escalar. La propia Academia reconoce esta
confusión en la definición de «pesar»: “Determinar el peso, o más propiamente, la masa de algo
por medio de la balanza o de otro instrumento equivalente”.

La masa de un cuerpo es una propiedad intrínseca del mismo, la cantidad de materia,


independiente de la intensidad del campo gravitatorio y de cualquier otro efecto. Representa la
inercia o resistencia del cuerpo a los cambios de estado de movimiento (aceleración, masa

50
inercial), además de hacerla sensible a los efectos de los campos gravitatorios (masa
gravitacional).

El peso de un cuerpo, en cambio, no es una propiedad intrínseca del mismo, ya que depende de
la intensidad del campo gravitatorio en el lugar del espacio ocupado por el cuerpo. La distinción
científica entre “masa” y “peso” no es importante para muchos efectos prácticos porque la
fuerza gravitatoria no experimenta grandes cambios en las proximidades de la superficie
terrestre. En un campo gravitatorio constante la fuerza que ejerce la gravedad sobre un cuerpo
(su peso) es directamente proporcional a su masa. Pero en realidad el campo gravitatorio
terrestre no es constante; puede llegar a variar hasta en un 0,5 % entre los distintos lugares de
la Tierra, lo que significa que se altera la relación “masa-peso” con la variación de la fuerza de
la gravedad.

Por el contrario, el peso de un mismo cuerpo experimenta cambios muy significativos al


cambiar el objeto masivo que crea el campo gravitatorio. Así, por ejemplo, una persona de 60
kg (6,118 UTM) de masa, pesa 588,60 N (60 kgf) en la superficie de la Tierra. La misma
persona, en la superficie de la Luna pesaría tan solo unos 98,05 N (10 kgf); sin embargo, su
masa seguirá siendo de 60 kg (6,118 UTM).

Bajo la denominación de peso aparente se incluyen otros efectos, además de la fuerza


gravitatoria y el efecto centrífugo, como la flotación, el carácter no inercial del sistema de
referencia (v.g., un ascensor acelerado), etc. El peso que mide el dinamómetro, es en realidad
el peso aparente; el peso real sería el que mediría en el vacío en un referencial inercial.

Unidades de peso
Como el peso es una fuerza, se mide en unidades de fuerza. Sin embargo, las unidades de peso
y masa tienen una larga historia compartida, en parte porque su diferencia no fue bien entendida
cuando dichas unidades comenzaron a utilizarse.

Sistema Internacional de Unidades


Este sistema es el prioritario o único legal en la mayor parte de las naciones (excluidas Birmania
y Estados Unidos), por lo que en las publicaciones científicas, en los proyectos técnicos, en las
especificaciones de máquinas, etc., las magnitudes físicas se expresan en unidades del Sistema
Internacional de Unidades (SI). Así, el peso se expresa en unidades de fuerza del SI, esto es, en
newtons (N):

1 N = 1 kg · 1 m/s²

Sistema Técnico de Unidades


En el Sistema Técnico de Unidades, el peso se mide en kilogramo-fuerza (kgf) o kilopondio
(kp), definido como la fuerza ejercida sobre un kilogramo de masa por la aceleración en caída
libre (g = 9,80665 m/s²)

1 kp = 9,80665 N = 9,80665 kg·m/s²

Otros sistemas
También se suele indicar el peso en unidades de fuerza de otros sistemas, como la dina, la libra-
fuerza, la onza-fuerza, etcétera.

51
La dina es la unidad CGS de fuerza y no forma parte del SI. Algunas unidades inglesas, como
la libra, pueden ser de fuerza o de masa. Las unidades relacionadas, como el slug, forman parte
de sub-sistemas de unidades.

Unidad de Medida de Trabajo


Para el sistema MKS es la unidad de medida del trabajo en el Sistema Internacional es el Julio o Joule
(J). Un Julio es el trabajo que realiza una fuerza constante de 1 Newton sobre un cuerpo que se
desplaza 1 metro en la misma dirección y sentido que la fuerza.
Para el sistema CGS la unidad de medida de trabajo es el ERGIO, que es igual a DINA x cm. y para el

Unidades de medida del calor


El calor es una forma de energía, y sus unidades de medida son el Joule (J) y la caloría (cal) (1
cal = 4,186 J) que fue definida en su momento para el calor cuando no se había establecido
que era una forma de energía.
Caloría: Es la cantidad de calor que debe extraerse o transferirse a un gramo de agua para
cambiar su temperatura en 1º C (cambiar su temperatura significa aumentarla en 1º C o
disminuirla en lº C). Se abrevia “cal”.

Junto con la caloría se usa también la kilocaloría para medir el calor.

Kilocaloría: Es la cantidad de calor que debe extraerse o transferirse a 1 kilogramo de agua


para cambiar su temperatura en 1º C. Se abrevia kcal.

Unidad de Medida de la Presión


En el sistema internacional (SI) la unidad de fuerza es el Newton y la superficie se expresa en
metros cuadrados la unidad de presión será:
p = F/A = 1 newton/1 metro cuadrado = 1 Pascal
1 Pa =1N/m2

El Pa es una unidad de presión bastante pequeña por lo cual para los valores que utilizamos en
química se emplea un múltiplo el kilopascal (kPa) que es mil veces mayor.

Debido a que la presión se mide muchas veces en relación con la de un líquido no es de extrañar
que las unidades de presión que todavía se emplean con frecuencia, a pesar de la recomendación
en favor del SI, sean medidas de longitud de líquidos ya que siendo constante la gravedad y la
densidad del líquido utilizado, la presión depende solo de la altura de la columna de líquido.

Dentro de estas tenemos dos, una de utilización frecuente en el lado arterial del sistema
cardiovascular, el milímetro de mercurio, la otra se utiliza para presiones en la parte venosa de
la circulación general y en las presiones dentro del sistema respiratorio, el centímetro de agua.
mm de Hg es la presión que ejerce una columna de mercurio (densidad a 0º 13,595 g/cm3)
de 1 mm de altura para una aceleración de la gravedad de 9,80665 ms-2 En honor de Torricelli
se denomina torr.
cm de H2O es la presión que ejerce una columna de agua (densidad a 0º 1 g/cm3) de 1 cm de
altura. Equivale en mm de Hg a algo menos de un torr:
g*h*13,595=g*10*1
h=10/13,595= 0,736 mm

52
Si consideramos que la presión de un gas con la que convivimos desde que nacemos es la
presión que ejerce el aire (es una mezcla de gases) sobre nosotros no es de extrañar que una
buena parte de las medidas de presión se relacionen con la atmósfera.

En el siglo XVII Torricelli descubrió que si llenaba un tubo de mercurio y tapando el extremo
abierto lo introducía en una cuba con mercurio, al destapar dicho extremo el nivel de mercurio
descendía pero sólo hasta determinada altura y luego se mantenía a ese nivel. La explicación de
porqué se mantiene así es que el aire ejerce una presión sobre la cubeta que se transmite a todo
el mercurio pero como en el espacio por encima del tubo no hay aire en esa zona no ejerce
efecto la presión atmosférica y por ello empuja al mercurio hasta que su altura equilibra a la
presión. El instrumento que permite, por lo tanto, medir la presión atmosférica se
denomina barómetro de mercurio.

Atmósfera estándar. La presión atmosférica varia con las condiciones meteorológicas y con
la altitud. La atmósfera estándar es la presión que ejerce una columna de mercurio con una
altura de 760 mm (densidad 13,5951 g/cm3 a 0ºC, g=9,80665 ms-2 es decir a 45º de latitud y al
nivel del mar). La relación entre una atmósfera de presión y la unidad del SI, el pascal es:
1 atm = 760 mmHg = 0,76 m *13.595 kg m-3 * 9,80665 ms-2 = 101325 Pa =101,325 kPa
1 mm Hg = 101325/760 = 133,3 Pa

bar El bar es un múltiplo del Pa, equivale a 100 kPa. Se utiliza un submúltiplo del bar el mbar,
que equivale a 100 Pa (es decir, un hectopascal hPa). Se emplea en meteorología. La presión
de una atmósfera corresponde por lo tanto a 101325/100 = 1013,25 mb. El bar equivale a 750
mmHg, 10,197 mH2O y 0,987 atm

Estándares
Por acuerdo internacional, los estándares se fijan arbitrariamente para las cantidades de masa,
longitud, tiempo, temperatura y mol. Éstas son cinco de las unidades base del SI.
A continuación se mencionan los estándares comúnmente utilizados.
El estándar de masa es el kilogramo (kg), definido como la masa del kilogramo internacional,
es un cilindro de platino que se conserva en Sèvres, Francia.
El estándar de longitud es el metro (m), definido (desde 1983) como la longitud de onda de la
ruta recorrida por la luz en el vacío durante un periodo de 1/299 792 458 de un segundo.
El estándar de tiempo es el segundo (s), definido como 9 192 631.770 ciclos de frecuencia de
una cierta transición cuántica de un átomo de Ce.
El estándar de temperatura es el Kelvin (K), que se define asignando el valor de 273.16 K a la
temperatura del agua pura en su punto triple, la única temperatura a la que el agua líquida, el
hielo y el vapor de agua coexisten en equilibrio.
El mol (abreviado de igual forma) se define como la cantidad de una sustancia que contiene
tantas unidades primarias como átomos hay en 12 g de C. La definición de mol es equivalente
a la afirmación de que la masa de un mol de una sustancia pura en gramos es numéricamente
igual a su peso molecular calculado a partir de la tabla estándar de pesos atómicos, en la cual el
peso atómico del C está dado por 12.01115. Este número difiere de 12 porque se aplica a la
mezcla isotópica natural del carbono en lugar de hacerlo al C puro. En los cálculos de ingeniería,
los términos kilogramo mol y libra mol se utilizan comúnmente para designar la masa de una
sustancia pura en kilogramos o libras que sean iguales a su peso molecular.
El número de moléculas en un gramo mol está dado por el número de Avogadro, 6.022x1023

Evaluación de constantes

53
A partir de los estándares básicos, se miden los valores de m, ma y mb de las ecuaciones (1.1) y
(1.2) en kilogramos, r en metros y u en metros por segundo. Las constantes k1 y k2 no son
independientes sino que están relacionadas entre sí eliminando F de las ecuaciones (1.1) y (1.2).
De esta forma se obtiene:

Tanto k1 como k2 se pueden fijar en forma arbitraria. Por lo tanto, es preciso obtener las demás
constantes por medio de experimentos en los cuales las fuerzas de inercia calculadas por la
ecuación (1.1) se comparan con las fuerzas de gravitación calculadas por la ecuación (1.2).
En el sistema SI, k1 se fija como la unidad y k2 se obtiene experimentalmente.
Entonces la ecuación (1.1) se convierte en:

F = ma
La fuerza definida por la ecuación (1.5), también usada en la ecuación (1.2), se denomina
newton (N). A partir de la ecuación (1.5)

La constante k2 se representa por G y se denomina constante de gravitación. Su valor


recomendado es

Trabajo, energía y potencia


En el sistema SI, tanto el trabajo como la energía se miden en newton-metros, una unidad
llamada joule (J), y de esta manera

La potencia se mide en joules por segundo, una unidad llamada watt (W).

Calor
Es posible fijar la constante k3 de la ecuación (1.3) en forma arbitraria. En el sistema SI, se
considera al igual que k1, como la unidad. La ecuación (1.3) se convierte en:

El calor, al igual que el trabajo, se mide en joules.

Unidades decimales
En el sistema SI, se define una sola unidad para cada magnitud, pero también se reconocen
múltiplos y submúltiplos decimales con nombres propios. Están listados en la tabla 1.1. El
tiempo puede expresarse en unidades no decimales: minutos (min), horas (h) o días (d).

Gravedad estándar
Para ciertos propósitos, se utiliza la aceleración de la caída libre en el campo gravitacional de
la Tierra, esta magnitud, que se representa por g, es casi constante. Varía ligeramente en función
de la latitud y la altura sobre el nivel del mar. Para cálculos precisos, se ha establecido un
estándar arbitrario gn definido por

Unidades de presión

54
La unidad de presión en el sistema SI es el newton por metro cuadrado. Esta unidad, llamada
pascal (Pa), es muy pequeña, por lo que también se utiliza un múltiplo, llamado bar, definido
por

Una unidad de presión empírica más común, utilizada con todos los sistemas de unidades, es
la atmósfera estándar (atm), definida como

Unidades cgs
El sistema más antiguo centímetro-gramo-segundo (cgs) se puede derivar del sistema SI
tomando ciertas decisiones arbitrarias.
El estándar para la masa es el gramo (g), definido por

El estándar para la longitud es el centímetro (cm), definido como

Los estándares de tiempo, temperatura y mol no se modifican.


Como en el SI, la constante k1 de la ecuación (1.1) se fija como la unidad. La unidad de fuerza
recibe el nombre de dina (din), definida por

La unidad de energía y trabajo es el ergio (erg), definido por

La constante k3 en la ecuación (1.3) no es una unidad. Una unidad de calor, llamada caloría
(cal), se utiliza para convertir en ergios la unidad de calor. La constante 1/k3 se sustituye por J,
que representa el llamado equivalente mecánico del calor y se mide en joules por caloría. La
ecuación (1.3) se convierte en

Se han definido dos calorías diferentes. La caloría termoquímica (cal), utilizada en química,
termodinámica de ingeniería química y cinética de reacción, se define por

La caloría de las tablas internacionales del vapor de agua (calIT), usada en la ingeniería de
potencia de calor, se define como

La caloría también se define de tal forma que el calor específico del agua es aproximadamente
1 cal/g. °C.
La aceleración estándar de la caída libre en unidades cgs, es

Unidades de ingeniería fps

55
En algunos países se ha utilizado ampliamente, tanto en actividades comerciales como en
ingeniería, el sistema gravitacional de unidades no decimal. El sistema puede derivarse del SI
considerando las decisiones siguientes.
El estándar de masa es la libra (lb), definida como

El estándar de longitud es la pulgada (in.) definida como 2.54 cm. Esto es equivalente a definir
el pie (ft) como

El estándar de tiempo sigue siendo el segundo (s).


La escala termodinámica de temperatura recibe el nombre de escala Rankine, en la que las
temperaturas se representan por grados Rankine y se definen como

El punto de congelación del agua en la escala Rankine es 273.15 x 1.8 = 491.67 °R.
La escala análoga de la Celsius es la escala Fahrenheit, en la que las lecturas se representan por
grados Fahrenheit. Se deriva de la escala Rankine, tomando su punto cero exactamente 32 °F
por debajo del punto de congelación del agua en la escala Rankine, de tal manera que

La relación entre las escalas Celsius y Fahrenheit se proporciona por la ecuación exacta
siguiente:

A partir de esta ecuación, las diferencias de temperatura se relacionan por

El punto de evaporación del agua es 212.00 °F.

Libra fuerza
El sistema fps se caracteriza por una unidad gravitacional de fuerza, llamada libra fuerza (lbf).
La unidad se define de tal forma que el campo gravitacional estándar ejerce una fuerza de una
libra sobre la masa de una libra. La aceleración estándar de la caída libre en unidades fps, con
cinco cifras significativas, es

La libra fuerza se define por

Entonces la ecuación (1.1) resulta

También es posible escribir la ecuación (1.1) con 1/gc en lugar de k1:

56
La comparación entre las ecuaciones (1.34) y (1.35) muestra que para conservar tanto la
igualdad numérica como la consistencia de las unidades, es necesario definir gc, el llamado
factor de proporcionalidad de la ley de Newton para la unidad de fuerza gravitacional, por

La unidad de trabajo y de energía mecánica en el sistema fps es el pie-libra-fuerza (ft·lbf). La


potencia se mide por una unidad empírica, el caballo de fuerza (hp), definido por

La unidad de calor es la unidad británica térmica (Btu, por sus siglas en inglés), definida por la
relación implícita

Como en el sistema cgs, la constante k3 en la ecuación (1.3) se sustituye por 1/J, donde J es el
equivalente mecánico del calor, igual a 778.17 ft · lbf/Btu.
La definición de la Btu requiere que el valor numérico del calor específico sea el mismo en
ambos sistemas y que en cada caso el calor específico del agua sea aproximadamente 1.0.

Constante de los gases


Si la masa se mide en kilogramos o gramos, la constante k4 en la ecuación (1.4) difiere de un
gas a otro. Pero cuando se usa el concepto de mol como una unidad de masa, es posible sustituir
k4, que puede sustituirse por la constante universal de los gases R, que, según la ley de
Avogadro, es la misma para todos los gases. El valor numérico de R sólo depende de las
unidades elegidas para la energía, temperatura y masa. Entonces la ecuación (1.4) se escribe de
la manera siguiente:

Donde n es el número de moles. Esta ecuación se aplica también a mezclas de gases si n es el


número total de moles de todas las especies moleculares que forman el volumen V.

57
En la tabla 1.2 se proporcionan valores de R en otras unidades de energía, temperatura y masa.
Volumen molar estándar. De la tabla 1.2, el volumen de 1 kg mol de gas en condiciones
normales
(1 atm, 0°C), es 82.056 x10–3 x 273 = 22.4 m3 o 22.4 (L/g mol). En unidades fps, el volumen
estándar a 1 atm y 32 °F es 359 ft3 /lb mol.

Conversión de unidades
Puesto que comúnmente se utilizan tres sistemas de unidades, con frecuencia es necesario
convertir las magnitudes de las cantidades de un sistema a otro. Para ello se emplean factores
de conversión. Sólo se requieren los factores de conversión definidos para las unidades base,
ya que los factores de conversión para todas las demás unidades pueden calcularse a partir de
ellos. Las conversiones entre los sistemas cgs y SI son sencillas.
Ambos emplean los mismos estándares para el tiempo, la temperatura y el mol, y únicamente
se necesitan las conversiones decimales definidas por las ecuaciones (1.16) y (1.17). Los
sistemas SI y fps también utilizan el segundo como el estándar de tiempo.
De esta manera, los tres factores de conversión definidos para la masa, la longitud y la
temperatura por las ecuaciones (1.24), (1.25) y (1.26), respectivamente, son suficientes para
todas las conversiones de unidades entre estos dos sistemas.
El ejemplo 1.1 demuestra cómo se calculan los factores de conversión a partir de números
exactos utilizados para el establecimiento de las definiciones de las unidades de los sistemas SI
y fps. En las conversiones en las que interviene gc en unidades fps, se recomienda el uso de la
relación numérica exacta 9.80665/0.3048 en lugar del número fps 32.1740 con el fin de obtener
la precisión máxima en el cálculo final y tomar ventaja de cancelaciones posibles de números
durante el cálculo.

El análisis dimensional es una técnica a través de la cual se determinan todas las dimensiones
que se encuentran en las mismas unidades, para tal efecto se multiplica el número que se desea
transformar por un factor de conversión.

Las unidades se manejan como números, y si una misma unidad se encuentra en el numerador
y en el denominador éstas se cancelan. Los factores de conversión se utilizan para expresar la
equivalencia de una magnitud en unidades diferentes (1 cm = 10 mm).
Como el numerador y el denominador describen la misma cantidad, el factor es equivalente al
número 1. Por tanto, multiplicar por este factor no cambia la cantidad medida, sólo cambian las
unidades.

o de otra manera,

Cantidad deseada = cantidad dada × factor de conversión

Cuando se multiplica la cantidad dada por el factor de conversión apropiado, algunas de las
unidades se cancelan para dar el resultado deseado.
Es decir, para simplificar la conversión de una unidad a otra se usa la técnica denominada
método de
factor-unidad, la cual se fundamenta en el análisis dimensional.
Con este enfoque, cualquier problema que requiera conversión de una unidad a otra se puede
resolver de manera similar.

58
En la realización de estos cálculos es necesario contar con herramientas que muestren las
equivalencias entre unidades de medición (véase tabla 2.12).

59
Otra herramienta útil en la solución de problemas, en lo general y en lo particular de procesos
industriales, es el uso de números exponenciales, para tal efecto existen dos buenas razones:
1. Para evitar escribir gran cantidad de ceros en números muy grandes o pequeños.

60
2. Proporcionar una manera de expresar información empleando cifras significativas con
la mínima confusión.

La idea de los números exponenciales es simplificar las cantidades expresadas como múltiplos
de las potencias de 10. Así, un número exponencial, como 6,02 × 1023, consta de dos partes: un
coeficiente (en este caso 6,02) y una potencia de 10 (en este caso 1023).
El coeficiente es usualmente elegido para estar entre 1 y 10, el exponente en la potencia de 10
puede ser negativo o positivo. Un exponente positivo n significa multiplicar por 10 n veces
(esto es, moviendo las decimales n lugares a la derecha) y un exponente negativo m significa
dividir por 10 m veces (esto es, mover el punto decimal m veces hacia la izquierda).
La siguiente tabla lista las formas equivalentes de escribir algunos números sencillos

Potencias de 10 fraccionarias y decimales también son posibles, pero no pueden ser


interpretados en términos de añadir o eliminar ceros o mover la coma decimal.
Una de las grandes ventajas del SI es que éste es un sistema coherente, característica que se
resalta en los siguientes ejemplos.

61
Recomendaciones para la escritura de las unidades del SI
Junto con las definiciones de las unidades del SI se emitieron ciertas recomendaciones
tendientes a unificar en forma universal la escritura de dichas unidades, sus símbolos y otros
conceptos relacionados. Se recomienda escribir las cantidades de cuatro dígitos sin
separación alguna y eliminar la “coma” en los millares, aunque es aceptado una
separación entre cantidades: 1 000, 1 348, 130 899, etcétera.
Esta recomendación también es aplicable a los grupos de tres dígitos después de la
coma decimal: 0,234 5, 0,435 671, etcétera.

La coma decimal, usada en Europa, o el punto decimal, usado en Estados Unidos de América,
son aceptados, con las recomendaciones anteriormente citadas.
Los símbolos de las unidades no se deben escribir en plural, sino con la letra o las letras que lo
representan (véase tabla 2.13); en cambio, si se escribe el nombre completo de la unidad sí se
puede llevar a la forma plural según la regla del idioma español; escribiendo los nombres
completos se les considera nombres comunes y, por tanto, se escriben en minúsculas.
Cuando el símbolo de un múltiplo o de un submúltiplo de una unidad lleva exponente, éste
afecta no sólo a la parte del símbolo que designa la unidad, sino al conjunto del símbolo.

mm2 significa (mm)2, es el área que tiene un mm de lado.


Cabe resaltar que no se debe dejar espacio entre un prefijo y un símbolo:

mm y no m m cm y no c m dl y no d l

Si se combinan dos o más símbolos de unidades se recomiendan dejar un espacio entre ellos:
kW h; m kg

62
Se recomienda dejar un espacio entre la cifra que indica la medida y la unidad de medida:
53 kg; 28 mg
Cuando una unidad derivada se forme dividiendo una unidad por otra, se puede utilizar la
barra horizontal u oblicua o potencias negativas, nunca debe introducirse en una misma línea
más de una barra oblicua, debiéndose utilizar en estos casos potencias negativas o paréntesis.

A continuación se presenta un ejemplo de expresiones sinónimas:

En éstas se admiten los prefijos compuestos formados por la yuxtaposición de varios prefijos.
Por ejemplo, no se admiten 1 µµF ni 1 m µg; sino que se debe escribir 1 pF (picofaradio) y 1
ng (nanogramo).
Asimismo, no se debe poner punto al final de un símbolo como si se tratará de una abreviatura,
excepto cuando termina una frase:
25 cm 8,43 m 72,2 A
Todos los nombres de las unidades, aunque lleven el nombre de un científico, se escriben con
inicial minúscula y los símbolos de las unidades que corresponden al nombre de un científico
se escriben con mayúscula:
celsius °C, volt V, ampere A, coulomb C
El símbolo ºC para los grados Celsius, es precedido por un espacio cuando se expresa
el valor de la temperatura Celsius:
T = 23,4 ºC y no T = 23,4ºC o T = 23,4 ºC
No se deben mezclar nombres completos con símbolos en unidades compuestas:
kg m y no kilogramos m; A h y no ampere h.
La IX CGPM (1948) recomendó para la escritura de números que se agrupen las cifras de tres
en tres a partir de la coma, tanto hacia la derecha como hacia la izquierda, separando cada
grupo por un espacio y nunca por puntos.
Ejemplo: el valor en centímetros de la longitud de onda de la radiación emitida por el átomo
de criptón 86 debe escribirse 16 507,637 3 y nunca debe escribirse 16.507,6373.

Se recomienda que el valor numérico que representa una medida esté comprendido entre uno y
mil, de esta manera no debe decirse: “la frecuencia de los ultrasonidos que es superior a 20 000
Hz, se debe indicar como superior a 20 K Hz”.
En general, las reglas para la escritura de los símbolos en el SI son:
➥ Los símbolos de las unidades del SI, a excepción del ohm Ω, se expresan en caracteres
romanos, en general, con minúsculas.
➥ Las unidades derivadas de nombres propios se expresan con su letra inicial en mayúscula.
➥ Los símbolos no van seguidos de punto, ni toman la “s” para el plural.
➥ Cuando el símbolo de un múltiplo o submúltiplo de una unidad lleva exponente, ésta afecta
no sólo a la parte del símbolo que designa la unidad, sino al conjunto del símbolo.
➥ El símbolo de la unidad sigue al símbolo del prefijo (sin espacio).
➥ El producto de los símbolos de dos o más unidades se indica de preferencia por medio de un
punto como símbolo de multiplicación.

63
➥ Cuando una unidad derivada sea el cociente de otras dos, se puede utilizar la barra oblicua
(/), la barra horizontal o bien potencias negativas para evitar el denominador.
➥ En una misma línea no se debe introducir más de una barra oblicua, a menos que se añadan
paréntesis, a fin de evitar toda ambigüedad. En los casos complejos pueden utilizarse paréntesis
o potencias negativas.
➥ Los nombres de las unidades debidos a nombres propios deben escribirse con idéntica
ortografía que el nombre de éstos, pero con minúscula inicial.
No obstante, serán igualmente aceptables sus denominaciones castellanizadas de uso habitual,
siempre que estén reconocidas por la Real Academia de la Lengua Española.
➥ Los nombres de las unidades toman una “s” en el plural, excepto las que terminan en s, x o
z.
➥ En los números, la coma se utiliza sólo para separar la parte entera del decimal. Para facilitar
la lectura, los números pueden estar divididos en grupos de tres cifras; estos grupos no se
separan por puntos ni comas. Las separaciones en grupos no se utilizan para los números de
cuatro cifras que designan un año.
➥ Para las unidades del SI derivadas, que se expresan como productos o cocientes, para indicar
división, se utiliza la preposición “por” entre los nombres de las unidades y para indicar
multiplicación no se utiliza ninguna palabra.

Metodología para resolver problemas de conversión de unidades


Al usar unidades dimensionales en la resolución de problemas, es recomendable seguir los
pasos que a continuación se describen:
1. Leer el problema.
2. Analizar el problema.
3. Anotar los datos del problema (generales y particulares).
4. Efectuar conversiones de unidades.
5. Obtener las respuestas a cada inciso.

Aplicación de la metodología para resolver problemas de conversión de


unidades
Los pasos descritos en la sección anterior se aplicarán al siguiente ejercicio referente a una
industria productora de champú.
En la planta productiva de una industria dedicada a la fabricación de champú se procesan 537
lb/h de producto con una densidad de 0,944 g/cm3
Determine:
a. La cantidad de galones procesados anualmente, considerando que la industria trabaja dos
turnos de 8 horas por día, cinco días por semana, cuatro semanas por mes, 12 meses al año.
b. ¿Cuántas botellas de 260 mL se requieren comprar cada seis meses? Considere que cada
botella se llena al 90% de su capacidad.
c. Si cada botella tiene un valor de $2,50, ¿cuál será la inversión mensual, bimestral y anual
por este concepto?
Lectura del problema
En este punto se observa que se proporcionan datos acerca del producto elaborado, la
producción obtenida, la jornada laboral establecida, la capacidad del recipiente y su costo.
Cada inciso está relacionado con la producción realizada por hora de trabajo; asimismo, se
deben efectuar algunas conversiones; al respecto se puede usar la técnica conocida como regla
de tres (uso de proporciones) o la conocida como conversión en línea (en la mayoría de los
problemas en que intervienen relaciones, existen cantidades dimensionales que es necesario
cancelar o que tienen equivalencias entre ellas). Este último es un método que depende de la

64
anulación de unidades dimensionales que aparecen tanto en el numerador como en el
denominador.

Análisis del problema


La frase: “en una industria productora de champú se procesan 537 lb/h de producto”, indica la
sustancia y la masa con la cual trabaja la empresa. La relación de materia procesada está descrita
como un gasto másico (cantidad de masa respecto al tiempo):

esto se entiende como la masa procesada por cada hora de producción, por lo que para conocerla
en forma diaria, semanal, etc., es necesario calcular la jornada laboral con la conversión
correspondiente.
Un dato adicional es la densidad, que es una propiedad que relaciona la masa y el volumen del
producto, esto corresponde a obtener las siguientes expresiones:

Estas relaciones serán utilizadas para llevar a cabo las conversiones correspondientes para
resolver correctamente los incisos contenidos en el problema.
La pregunta incluida en el inciso a) involucra la variable expresada en volumen (galones), y
por tanto debe ser utilizada la relación (1a) para obtener ese dato.
Siguiendo con la lectura del problema, la pregunta implica el cálculo del volumen (relación
con la densidad) en un determinado tiempo (cantidad de días laborables por año); también es
necesario obtener equivalencias para la masa y el volumen, incluidas en la relación de la
densidad.
Entonces, para resolver este inciso se expresan las cantidades de días laborables por año
(incluye las siguientes conversiones de tiempo):
1 turno = 8 horas
1 día = 2 turnos
1 semana = 5 días
1 mes = 4 semanas
1 año = 12 meses
y de masa y volumen:
1 libra = 454 g
1 gal = 3,84 L
1L = 1 000 mL
1 mL = 1 cm3
Para el inciso b) se requiere obtener la producción (por seis meses) y obtener el número de
botellas; para el primer cálculo, se aprovecha la respuesta obtenida en el inciso a), y para la
segunda hay que considerar que el llenado se efectúa al 90% de la capacidad de las botellas.
Finalmente, para el inciso c), ya calculado el número de botellas en b), se realiza un cociente
entre seis meses de trabajo para obtener la cantidad mensual de botellas utilizadas, esta cantidad
multiplicada por el precio unitario de cada botella indica el costo total por mes (multiplicado
por 2 es bimestral, por 6 es semestral y por 12 es anual).

Anotar los datos del problema


En este punto se escriben las referencias obtenidas en los pasos anteriores, resumiendo los datos
proporcionados en el problema:

65
a) Producto: champú

Galones por año:


1 turno = 8 horas
1 día = 2 turnos
1 semana = 5 días
1 año = 12 meses

La lista de datos para cada inciso, proporciona las incógnitas a resolver, las conversiones y la
secuencia de cálculos a realizar.

Efectuar conversiones de unidades


En este punto se usan conversiones que sean fácilmente recordables, ya que es más fácil
recordar que 1 L es igual a 1 000 mL, que 1 000 L son 106 cm3. Así, se inician los cálculos en
conversión en línea para los incisos:

y para las conversiones que involucran tiempo.

66
c) Continuando con los cálculos:

Obtener las respuestas de cada inciso


Las respuestas se pueden indicar en cada una de las conversiones de los incisos
correspondientes o creando una lista de respuestas como se muestra a continuación:
a) 258 260, 59 gal/año
b) 2 119 061 botellas/6 meses
c) $ 5 297 652,50 /mes
$10 595 305/bimestre
$ 63 571 830,99/año
De esta manera, el problema está completamente resuelto.

EJEMPLO 7 Generación de potencia eléctrica mediante una turbina eólica


Una escuela paga $0.09/kWh. Para reducir sus costos de energía, la escuela instala una turbina
de viento (Fig. 1-13) con una potencia nominal de 30 kW. Si la turbina trabaja 2 200 horas por
año a su potencia nominal, determine la cantidad de energía eléctrica generada por la turbina y
el dinero que ahorra la escuela por año.
Solución Se instala una turbina de viento (aerogenerador) para generar electricidad.
Se deben determinar la cantidad de energía eléctrica que se genera y el dinero que se ahorra por
año.
Análisis
La turbina de viento genera energía eléctrica a razón de 30 kW, o 30 kJ/s. Entonces, la
cantidad total de energía eléctrica generada por año es:
Energía total = (Energía por unidad de tiempo)(Intervalo de tiempo)
= (30 kW)(2 200 h)
= 66 000 kWh
La cantidad que se ahorra por año es el valor monetario de esta energía determinada como:
67
Dinero ahorrado = (Energía total)(costo unitario de energía)
= (66 000 kWh)($0.09/kWh)
= $5 940
Comentario La producción anual de energía eléctrica también se podría determinar en kJ
mediante manipulación de las unidades, como

Que es equivalente a 66 000 kWh (1 kWh = 3 600 kJ).

Se sabe por experiencia que las unidades pueden causar terribles dolores de cabeza si no se usan
con cuidado al resolver un problema. Sin embargo, con cierta atención y habilidad las unidades
se pueden usar de modo provechoso.
Sirven para comprobar fórmulas e incluso se pueden usar para deducir fórmulas, como se
explica en el siguiente ejemplo.

EJEMPLO 8 Obtención de fórmulas a partir de consideraciones de unidades


Se llena un depósito con aceite cuya densidad es 𝜌 = 850 kg/m3. Si el volumen del depósito
es 𝑉 = 2𝑚3, determine la cantidad de masa m en el depósito.
Solución Se tiene el volumen del depósito y se va a determinar la masa del aceite.
Suposiciones El aceite es una sustancia no compresible y, por lo tanto, su densidad es
constante.
Análisis Un bosquejo del sistema descrito se presenta en la figura 1-14.
Suponga que olvida la fórmula que relaciona la masa con la densidad y el volumen; sin
embargo, se sabe que la unidad de la masa es el kilogramo. Es decir, sin importar los cálculos
que se realicen se debe obtener al final la unidad de kilogramos.
Entendiendo mejor la información proporcionada, se tiene

Es evidente que se puede eliminar m3 y finalizar con kg al multiplicar estas dos cantidades.
Por lo tanto, la fórmula que se busca debe ser Así,

Así
68
Comentario Observe que existe la posibilidad de que este enfoque no funcione para fórmulas
más complejas. Las fórmulas constantes no dimensionales pueden estar presentes también, y
éstas no se pueden deducir sólo por consideraciones de unidades.

Relaciones de conversión de unidades


Así cómo es posible formar dimensiones no primarias mediante combinaciones adecuadas de
dimensiones primarias, todas las unidades no primarias (unidades secundarias) se forman a
través de combinaciones de unidades primarias.
Las unidades de fuerza, por ejemplo, es posible expresarlas como

Asimismo, se pueden expresar de modo más conveniente como relaciones de conversión de


unidades como

Las relaciones de conversión de unidades son iguales a 1 y no tienen unidades; por lo tanto,
tales relaciones (o sus inversos) se pueden insertar deforma conveniente en cualquier cálculo
para convertir unidades de manera adecuada (Fig. 1-16). Se recomienda a los estudiantes que
siempre usen relaciones de conversión de unidades. Algunos libros incluyen en las ecuaciones
la constante gravitacional arcaica gc definida como

con la finalidad de que concuerden las unidades de fuerza. Esta práctica produce una confusión
innecesaria y los autores de este libro consideran que no es aconsejable. En cambio, se
recomienda que los estudiantes usen relaciones de conversión de unidades.

69
EJEMPLO 9 El peso de una libra-masa
Por medio de las relaciones de conversión de unidades, muestre que 1.00 lbm pesa 1.00 lbf en
la Tierra (Fig. 1-17).
Solución Una masa de 1.00 lbm se somete a la gravedad terrestre estándar.
Se determinará su peso en lbf.
Suposiciones Se consideran condiciones estándar al nivel del mar.
Propiedades La constante gravitacional es g = 32.174 pie/s2
Análisis Se aplica la segunda ley de Newton para calcular el peso (fuerza) que corresponde a
la masa y aceleración conocidas. El peso de cualquier objeto es igual a su masa multiplicada
por el valor local de la aceleración debida a la gravedad. Así,

Explicación La cantidad dentro del paréntesis grande de esta ecuación es una relación de
conversión unitaria. La masa es la misma sin importar su ubicación. Sin embargo, en algún otro
planeta cuyo valor de la aceleración gravitacional es diferente, el peso de 1 lbm diferiría del
valor calculado aquí.

Cuando se compra una caja de cereal en la impresión se lee “Peso neto: una libra (454 gramos)”
(Fig. 1-18). En términos técnicos, esto significa que el cereal dentro de la caja pesa 1.00 lbf en
la Tierra y tiene una masa de 453.6 g (0.4536 kg). Usando la segunda ley de Newton, el peso
real del cereal en el sistema métrico es

70
EJEMPLO 10 Utilizando sólo definiciones y estándares exactos, calcule factores para
convertir
a. Newtons a libras fuerza,
b. unidades británicas térmicas Btu a calorías IT,
c. atmósferas a libras fuerza por pulgada cuadrada y
d. caballo de fuerza a kilowatts.
Solución
a) A partir de las ecuaciones (1.6), (1.24) y (1.25),

A partir de la ecuación (1.30)

Y así

En el apéndice 1 se muestra que para convertir newtons a libras fuerza, es preciso


multiplicar por 0.224809. Obviamente, para convertir libras fuerza a newtons hay que
multiplicar por 1/0.224809 = 4.448221.

b) A partir de la ecuación (1.36)

A partir de las ecuaciones (1.16), (1.24) y (1.29)

c) A partir de las ecuaciones (1.6), (1.14) y (1.15)


1 atm = 1.01325 x 105 kg · m/s2 . m2
A partir de las ecuaciones (1.24), (1.25) y (1.35), puesto que 1 ft = 12 in.,

71
d) A partir de las ecuaciones (1.31) y (1.35)

Utilizando las ecuaciones (1.24) y (1.25) resulta

Sustituyendo en la ecuación (1.8) y dividiendo entre 1 000,


1 hp = 0.74570 kW.
Aunque es posible calcular los factores de conversión cuando se necesitan, es más
eficiente usar tablas de los factores más comunes. En el apéndice 1 se presenta una
tabla de los factores utilizados en este curso.

1 Actividad propuesta
A. Para cada una de las siguientes magnitudes escriba el nombre de su unidad dimensional
correspondiente en el SI y localícela en la sopa de letras anexa.
1. Longitud
2. Masa
3. Tiempo
4. Intensidad de corriente eléctrica
5. Temperatura termodinámica
6. Cantidad de sustancia
7. Intensidad luminosa
8. Ángulo plano
9. Ángulo sólido
10. Frecuencia
11. Fuerza
12. Presión
13. Energía
14. Potencia
15. Potencial eléctrico
16. Diferencia de potencial
17. Capacitancia
18. Resistencia eléctrica
19. Conductancia
20. Flujo magnético
21. Densidad de flujo magnético
22. Inductancia
23. Temperatura cotidiana
24. Flujo luminoso
25. Iluminación

B. Masa, fuerza y unidades

72
73

S-ar putea să vă placă și